Bài tập đại số 10 nguyễn công nhựt

112 953 0
Bài tập đại số 10 nguyễn công nhựt

Đang tải... (xem toàn văn)

Tài liệu hạn chế xem trước, để xem đầy đủ mời bạn chọn Tải xuống

Thông tin tài liệu

NGUY N CÔNG NH T ---- ›š & ›š ---- BAØI TAÄP OÂN THI TOÁT NGHIEÄP THPT & ÑAÏI HOÏC Naêm 2014 Mệnh đề – Tập hợp CHƯƠNG I MỆNH ĐỀ – TẬP HỢP I. MỆNH ĐỀ 1. Mệnh đề · Mệnh đề là một câu khẳng định đúng hoặc một câu khẳng định sai. · Một mệnh đề không thể vừa đúng, vừa sai. 2. Mệnh đề phủ định Cho mệnh đề P. · Mệnh đề "Không phải P" đgl mệnh đề phủ định của P và kí hiệu là P . · Nếu P đúng thì P sai, nếu P sai thì P đúng. 3. Mệnh đề kéo theo Cho hai mệnh đề P và Q. · Mệnh đề "Nếu P thì Q" đgl mệnh đề kéo theo và kí hiệu là P Þ Q. · Mệnh đề P Þ Q chỉ sai khi P đúng và Q sai. Chú ý: Các định lí toán học thường có dạng P Þ Q. Khi đó: – P là giả thiết, Q là kết luận; – P là điều kiện đủ để có Q; – Q là điều kiện cần để có P. 4. Mệnh đề đảo Cho mệnh đề kéo theo P Þ Q. Mệnh đề Q Þ P đgl mệnh đề đảo của mệnh đề P Þ Q. 5. Mệnh đề tương đương Cho hai mệnh đề P và Q. · Mệnh đề "P nếu và chỉ nếu Q" đgl mệnh đề tương đương và kí hiệu là P Û Q. · Mệnh đề P Û Q đúng khi và chỉ khi cả hai mệnh để P Þ Q và Q Þ P đều đúng. Chú ý: Nếu mệnh đề P Û Q là một định lí thì ta nói P là điều kiện cần và đủ để có Q. 6. Mệnh đề chứa biến Mệnh đề chứa biến là một câu khẳng định chứa biến nhận giá trị trong một tập X nào đó mà với mỗi giá trị của biến thuộc X ta được một mệnh đề. 7. Kí hiệu " và $ · ""x Î X, P(x)" · "$x Î X, P(x)" · Mệnh đề phủ định của mệnh đề ""x Î X, P(x)" là "$x Î X, P( x ) ". · Mệnh đề phủ định của mệnh đề "$x Î X, P(x)" là ""x Î X, P( x ) ". 8. Phép chứng minh phản chứng Giả sử ta cần chứng minh định lí: A Þ B. Cách 1: Ta giả thiết A đúng. Dùng suy luận và các kiến thức toán học đã biết chứng minh B đúng. Cách 2: (Chứng minh phản chứng) Ta giả thiết B sai, từ đó chứng minh A sai. Do A không thể vừa đúng vừa sai nên kết quả là B phải đúng. 9. Bổ sung Cho hai mệnh đề P và Q. · Mệnh đề "P và Q" đgl giao của hai mệnh đề P và Q và kí hiệu là P Ù Q. Mệnh đề P Ù Q đúng khi và chỉ khi cả P và Q đều đúng. · Mệnh đề "P hoặc Q" đgl hợp của hai mệnh đề P và Q và kí hiệu là P Ú Q. Mệnh đề P Ú Q sai khi và chỉ khi cả P và Q đều sai. · Phủ định của giao, hợp hai mệnh đề: P Ù Q = P Ú Q , PÚQ = P ÙQ. Trang 1 Mệnh đề – Tập hợp Nguy n Công Nh t Baøi 1. Trong các câu dưới đây, câu nào là mệnh đề, câu nào là mệnh đề chứa biến: a) Số 11 là số chẵn. c) Huế là một thành phố của Việt Nam. b) Bạn có chăm học không ? d) 2x + 3 là một số nguyên dương. e) 2 - 5 < 0 . g) Hãy trả lời câu hỏi này!. f) 4 + x = 3. h) Paris là thủ đô nước Ý. i) Phương trình x 2 - x + 1 = 0 có nghiệm. k) 13 là một số nguyên tố. ĐS: Mệnh đề: a, c, e, h, i, k. Mệnh đề chứa biến: d, f. Baøi 2. Trong các mệnh đề sau, mệnh đề nào là đúng ? Giải thích ? a) Nếu a chia hết cho 9 thì a chia hết cho 3. b) Nếu a ³ b thì a2 ³ b2 . c) Nếu a chia hết cho 3 thì a chia hết cho 6. d) Số p lớn hơn 2 và nhỏ hơn 4. e) 2 và 3 là hai số nguyên tố cùng nhau. f) 81 là một số chính phương. g) 5 > 3 hoặc 5 < 3. h) Số 15 chia hết cho 4 hoặc cho 5. ĐS: Mệnh đề đúng: a, d, e, f, g, h. Baøi 3. Trong các mệnh đề sau, mệnh đề nào là đúng ? Giải thích ? a) Hai tam giác bằng nhau khi và chỉ khi chúng có diện tích bằng nhau. b) Hai tam giác bằng nhau khi và chỉ khi chúng đồng dạng và có một cạnh bằng nhau. c) Một tam giác là tam giác đều khi và chỉ khi chúng có hai đường trung tuyến bằng nhau và có một góc bằng 600 . d) Một tam giác là tam giác vuông khi và chỉ khi nó có một góc bằng tổng của hai góc còn lại. e) Đường tròn có một tâm đối xứng và một trục đối xứng. f) Hình chữ nhật có hai trục đối xứng. g) Một tứ giác là hình thoi khi và chỉ khi nó có hai đường chéo vuông góc với nhau. h) Một tứ giác nội tiếp được đường tròn khi và chỉ khi nó có hai góc vuông. ĐS: Mệnh đề đúng: c, d, f. Baøi 4. Trong các mệnh đề sau, mệnh đề nào là đúng? Giải thích? Phát biểu các mệnh đề đó thành lời: a) "x Î R, x 2 > 0 . b) $x Î R, x > x 2 c) $x Î Q,4 x 2 - 1 = 0 . d) "n Î N , n2 > n . e) "x Î R, x 2 - x + 1 > 0 f) "x Î R, x 2 > 9 Þ x > 3 g) "x Î R, x > 3 Þ x 2 > 9 . h) "x Î R, x 2 < 5 Þ x < 5 i) $x Î R,5 x - 3 x 2 £ 1 k) $x Î N , x 2 + 2 x + 5 là hợp số. l) "n Î N , n2 + 1 không chia hết cho 3. m) "n Î N * , n(n + 1) là số lẻ. n) "n Î N * , n(n + 1)(n + 2) chia hết cho 6. ĐS: Mệnh đề đúng: b, c, e, g, i, k, l, n. Baøi 5. Điền vào chỗ trống từ nối "và" hay "hoặc" để được mệnh đề đúng: a) p < 4....p > 5 . b) ab = 0 khi a = 0.... b = 0 . c) ab ¹ 0 khi a ¹ 0.... b ¹ 0 d) ab > 0 khi a > 0.... b > 0.... a < 0.... b < 0 . e) Một số chia hết cho 6 khi và chỉ khi nó chia hết cho 2 …. cho 3. f) Một số chia hết cho 5 khi và chỉ khi chữ số tận cùng của nó bằng 0 …. bằng 5. ĐS: a) hoặc b) hoặc c) và d) và - hoặc - và e) và f) hoặc. Baøi 6. Cho mệnh đề chứa biến P(x), với x Î R. Tìm x để P(x) là mệnh đề đúng: a) P( x ) : " x 2 - 5 x + 4 = 0" b) P( x ) : " x 2 - 5 x + 6 = 0" c) P( x ) : " x 2 - 3 x > 0" d) P( x ) : " x ³ x " e) P( x ) : "2 x + 3 £ 7" f) P( x ) : " x 2 + x + 1 > 0" ĐS: a) x = 1; x = 4 b) x = 2; x = 3 c) x < 0 Ú x > 3 d) 0 £ x £ 1 e) x £ 2 f) x Î R . Baøi 7. Nêu mệnh đề phủ định của các mệnh đề sau: a) Số tự nhiên n chia hết cho 2 và cho 3. b) Số tự nhiên n có chữ số tận cùng bằng 0 hoặc bằng 5. c) Tứ giác T có hai cạnh đối vừa song song vừa bằng nhau. Trang 2 Mệnh đề – Tập hợp d) Số tự nhiên n có ước số bằng 1 và bằng n. Baøi 8. Nêu mệnh đề phủ định của các mệnh đề sau: a) "x Î R : x 2 > 0 . b) $x Î R : x > x 2 . c) $x Î Q : 4 x 2 - 1 = 0 . d) "x Î R : x 2 - x + 7 > 0 . e) "x Î R : x 2 - x - 2 < 0 . f) $x Î R : x 2 = 3 . g) "n Î N , n2 + 1 không chia hết cho 3. h) "n Î N , n2 + 2n + 5 là số nguyên tố. i) "n Î N , n2 + n chia hết cho 2. k) "n Î N , n2 - 1 là số lẻ. Baøi 9. Phát biểu các mệnh đề sau, bằng cách sử dụng thuật ngữ "điều kiện cần", "điều kiện đủ": a) Nếu một số tự nhiên có chữ số tận cùng là chữ số 5 thì nó chia hết cho 5. b) Nếu a + b > 0 thì một trong hai số a và b phải dương. c) Nếu một số tự nhiên chia hết cho 6 thì nó chia hết cho 3. d) Nếu a = b thì a2 = b2 . e) Nếu a và b cùng chia hết cho c thì a + b chia hết cho c. Baøi 10. Phát biểu các mệnh đề sau, bằng cách sử dụng thuật ngữ "điều kiện cần", "điều kiện đủ": a) Trong mặt phẳng, nếu hai đường thẳng phân biệt cùng vuông góc với một đường thẳng thứ ba thì hai đường thẳng ấy song song với nhau. b) Nếu hai tam giác bằng nhau thì chúng có diện tích bằng nhau. c) Nếu tứ giác T là một hình thoi thì nó có hai đường chéo vuông góc với nhau. d) Nếu tứ giác H là một hình chữ nhật thì nó có ba góc vuông. e) Nếu tam giác K đều thì nó có hai góc bằng nhau. Baøi 11. Phát biểu các mệnh đề sau, bằng cách sử dụng thuật ngữ "điều kiện cần và đủ": a) Một tam giác là vuông khi và chỉ khi nó có một góc bằng tổng hai góc còn lại. b) Một tứ giác là hình chữ nhật khi và chỉ khi nó có ba góc vuông. c) Một tứ giác là nội tiếp được trong đường tròn khi và chỉ khi nó có hai góc đối bù nhau. d) Một số chia hết cho 6 khi và chỉ khi nó chia hết cho 2 và cho 3. e) Số tự nhiên n là số lẻ khi và chỉ khi n2 là số lẻ. Baøi 12. Chứng minh các mệnh đề sau bằng phương pháp phản chứng: a) Nếu a + b < 2 thì một trong hai số a và b nhỏ hơn 1. b) Một tam giác không phải là tam giác đều thì nó có ít nhất một góc nhỏ hơn 600 . c) Nếu x ¹ -1 và y ¹ -1 thì x + y + xy ¹ -1 . d) Nếu bình phương của một số tự nhiên n là một số chẵn thì n cũng là một số chẵn. e) Nếu tích của hai số tự nhiên là một số lẻ thì tổng của chúng là một số chẵn. f) Nếu một tứ giác có tổng các góc đối diện bằng hai góc vuông thì tứ giác đó nội tiếp được đường tròn. g) Nếu x 2 + y 2 = 0 thì x = 0 và y = 0. Trang 3 Mệnh đề – Tập hợp Nguy n Công Nh t II. TẬP HỢP 1. Tập hợp · Tập hợp là một khái niệm cơ bản của toán học, không định nghĩa. · Cách xác định tập hợp: + Liệt kê các phần tử: viết các phần tử của tập hợp trong hai dấu móc { … }. + Chỉ ra tính chất đăc trưng cho các phần tử của tập hợp. · Tập rỗng: là tập hợp không chứa phần tử nào, kí hiệu Æ. 2. Tập hợp con – Tập hợp bằng nhau · A Ì B Û ( "x Î A Þ x Î B ) + A Ì A, "A + Æ Ì A, "A + A Ì B, B Ì C Þ A Ì C · A = B Û ( A Ì B vaø B Ì A ) 3. Một số tập con của tập hợp số thực · N* Ì N Ì Z Ì Q Ì R · Khoảng: (a; b) = { x Î R a < x < b} ; (a; +¥) = { x Î R a < x} ; (-¥; b) = { x Î R x < b} · Đoạn: [a; b] = { x Î R a £ x £ b} · Nửa khoảng: [a; b) = { x Î R a £ x < b} ; (a; b] = { x Î R a < x £ b} ; [a; +¥) = { x Î R a £ x} ; (-¥; b] = { x Î R x £ b} 4. Các phép toán tập hợp · Giao của hai tập hợp: A Ç B = { x x Î A vaø x Î B} · Hợp của hai tập hợp: · Hiệu của hai tập hợp: Phần bù: A È B = { x x Î A hoaëc x Î B} A \ B = { x x Î A vaø x Ï B} Cho B Ì A thì C A B = A \ B . Baøi 1. Viết mỗi tập hợp sau bằng cách liệt kê các phần tử của nó: { } C = { x Î R (6 x 2 - 7 x + 1)( x 2 - 5x + 6) = 0} A = x Î R (2 x 2 - 5 x + 3)( x 2 - 4 x + 3) = 0 E = { x Î N x + 3 < 4 + 2 x vaø 5x - 3 < 4 x - 1} G = { x Î N x < 5} ĐS: { } B = x Î R ( x 2 - 10 x + 21)( x 3 - x ) = 0 D = { x Î Z 2 x 2 - 5 x + 3 = 0} F = { x Î Z x + 2 £ 1} H = { x Î R x 2 + x + 3 = 0} Baøi 2. Viết mỗi tập hợp sau bằng cách chỉ rõ tính chất đặc trưng cho các phần tử của nó: A = {0; 1; 2; 3; 4} B = {0; 4; 8; 12; 16} C = {-3 ; 9; - 27; 81} D = {9; 36; 81; 144} E = {2; 3; 5; 7; 11} F = {3; 6; 9; 12; 15} G = Tập tất cả các điểm thuộc đường trung trực của đoạn thẳng AB. H = Tập tất cả các điểm thuộc đường tròn tâm I cho trước và có bán kính bằng 5. ĐS: Baøi 3. Trong các tập hợp sau đây, tập nào là tập rỗng: A = { x Î Z x < 1} B = { x Î R x 2 - x + 1 = 0} Trang 4 { } C = x Î Q x2 - 4x + 2 = 0 Mệnh đề – Tập hợp { } D = x Î Q x2 - 2 = 0 E = { x Î N x 2 + 7 x + 12 = 0} F = { x Î R x 2 - 4 x + 2 = 0} ĐS: Tập rỗng: B, C, D, E. Baøi 4. Tìm tất cả các tập con, các tập con gồm hai phần tử của các tập hợp sau: A = {1, 2} B = {1, 2, 3} C = {a, b, c, d} { } D = { x Î R 2 x 2 - 5 x + 2 = 0} E = x Î Q x2 - 4x + 2 = 0 Baøi 5. Trong các tập hợp sau, tập nào là tập con của tập nào? a) A = {1, 2, 3} , B = { x Î N x < 4} , C = (0; + ¥) , D = { x Î R 2 x 2 - 7 x + 3 = 0} . b) A = Tập các ước số tự nhiên của 6; B = Tập các ước số tự nhiên của 12. c) A = Tập các hình bình hành; B = Tập các hình chữ nhật; C = Tập các hình thoi; D = Tập các hình vuông. d) A = Tập các tam giác cân; B = Tập các tam giác đều; C = Tập các tam giác vuông; D = Tập các tam giác vuông cân. ĐS: Baøi 6. Tìm A Ç B, A È B, A \ B, B \ A với: a) A = {2, 4, 7, 8, 9, 12}, B = {2, 8, 9, 12} b) A = {2, 4, 6, 9}, B = {1, 2, 3, 4} c) A = { x Î R 2 x 2 - 3 x + 1 = 0} , B = { x Î R 2 x - 1 = 1} . d) A = Tập các ước số tự nhiên của 12, B = Tập các ước số tự nhiên của 18. { } e) A = x Î R ( x + 1)( x - 2)( x 2 - 8 x + 15) = 0 , B = Tập các số nguyên tố có một chữ số. { } f) A = { x Î Z x 2 < 4} , B = x Î Z (5 x - 3 x 2 )( x 2 - 2 x - 3) = 0 . { } g) A = x Î N ( x 2 - 9)( x 2 - 5 x - 6) = 0 , B = { x Î N x laø soá nguyeân toá , x £ 5} . ĐS: Baøi 7. Tìm tất cả các tập hợp X sao cho: a) {1, 2} Ì X Ì {1, 2, 3, 4, 5}. b) {1, 2} È X = {1, 2, 3, 4}. c) X Ì {1, 2, 3, 4}, X Ì {0, 2, 4, 6, 8} ĐS: Baøi 8. Tìm các tập hợp A, B sao cho: a) AÇB = {0; 1; 2; 3; 4}, A\B = {–3; –2}, B\A = {6; 9; 10}. b) AÇB = {1; 2; 3}, A\B = {4; 5}, B\A = {6; 9}. ĐS: Baøi 9. Tìm A Ç B, A È B, A \ B, B \ A với: a) A = [–4; 4], B = [1; 7] b) A = [–4; –2], B = (3; 7] c) A = [–4; –2], B = (3; 7) d) A = (–¥; –2], B = [3; +¥) e) A = [3; +¥), B = (0; 4) f) A = (1; 4), B = (2; 6) ĐS: Baøi 10. Tìm A È B È C, A Ç B Ç C với: a) A = [1; 4], B = (2; 6), C = (1; 2) b) A = (–¥; –2], B = [3; +¥), C = (0; 4) c) A = [0; 4], B = (1; 5), C = (−3; 1] d) A = (−¥; 2], B = [2; +¥), C = (0; 3) e) A = (−5; 1], B = [3; +¥), C = (−¥; −2) ĐS: Baøi 11. Chứng minh rằng: a) Nếu A Ì B thì A Ç B = A. c) Nếu A È B = A Ç B thì A = B b) Nếu A Ì C và B Ì C thì (A È B) Ì C. d) Nếu A Ì B và A Ì C thì A Ì (B Ç C). Trang 5 Mệnh đề – Tập hợp Nguy n Công Nh t III. SỐ GẦN ĐÚNG – SAI SỐ 1. Số gần đúng Trong đo đạc, tính toán ta thường chỉ nhận được các số gần đúng. 2. Sai số tuyệt đối Nếu a là số gần đúng của số đúng a thì Da = a - a đgl sai số tuyệt đối của số gần đúng a. 3. Độ chính xác của một số gần đúng Nếu Da = a - a £ d thì a - d £ a £ a + d . Ta nói a là ssố gần đúng của a với độ chính xác d, và qui ước viết gọn là a = a ± d . 4. Sai số tương đối Sai số tương đối của số gần đúng a là tỉ số giữa sai số tuyệt đối và a , kí hiệu d a = Da a . · d a càng nhỏ thì độ chính xác của phép đo đạc hoặc tính toán càng lớn. · Ta thường viết d a dưới dạng phần trăm. 5. Qui tròn số gần đúng · Nếu chữ số ngay sau hàng qui tròn nhỏ hơn 5 thì ta chỉ việc thay thế chữ số đó và các chữ số bên phải nó bởi số 0. · Nếu chữ số ngay sau hàng qui tròn lớn hơn hay bằng 5 thì ta thay thế chữ số đó và các chữ số bên phải nó bởi số 0 và cộng thêm một đơn vị vào chữ số ở hàng qui tròn. Nhận xét: Khi thay số đúng bởi số qui tròn đến một hàng nào đó thì sai sô tuyệt đối của số qui tròn không vượt quá nửa đơn vị của hàng qui tròn. Như vậy, độ chính xác của số qui tròn bằng nửa đơn vị của hàng qui tròn. 6. Chữ số chắc Cho số gần đúng a của số a với độ chính xác d. Trong số a, một chữ số đgl chữ số chắc (hay đáng tin) nếu d không vượt quá nửa đơn vị của hàng có chữ số đó. Nhận xét: Tất cả các chữ số đứng bên trái chữ số chắc đều là chữ số chắc. Tất cả các chữ số đứng bên phải chữ số không chắc đều là chữ số không chắc. Baøi 1. a) Trang 6 Hàm số bậc nhất – bậc hai CHƯƠNG II HÀM SỐ BẬC NHẤT VÀ BẬC HAI I. HÀM SỐ 1. Định nghĩa · Cho D Ì R, D ¹ Æ. Hàm số f xác định trên D là một qui tắc đặt tương ứng mỗi số x Î D với một và chỉ một số y Î R. · x đgl biến số (đối số), y đgl giá trị của hàm số f tại x. Kí hiệu: y = f(x). · D đgl tập xác định của hàm số. · T = { y = f ( x ) x Î D} đgl tập giá trị của hàm số. 2. Cách cho hàm số · Cho bằng bảng · Cho bằng biểu đồ · Cho bằng công thức y = f(x). Tập xác định của hàm số y = f(x) là tập hợp tất cả các số thực x sao cho biểu thức f(x) có nghĩa. 3. Đồ thị của hàm số Đồ thị của hàm số y = f(x) xác định trên tập D là tập hợp tất cả các điểm M ( x; f ( x ) ) trên mặt phẳng toạ độ với mọi x Î D. Chú ý: Ta thường gặp đồ thị của hàm số y = f(x) là một đường. Khi đó ta nói y = f(x) là phương trình của đường đó. 4. Sự biến thiên của hàm số Cho hàm số f xác định trên K. · Hàm số y = f(x) đồng biến (tăng) trên K nếu "x1 , x2 Î K : x1 < x2 Þ f ( x1 ) < f ( x2 ) · Hàm số y = f(x) nghịch biến (giảm) trên K nếu "x1 , x2 Î K : x1 < x2 Þ f ( x1 ) > f ( x2 ) 5. Tính chẵn lẻ của hàm số Cho hàm số y = f(x) có tập xác định D. · Hàm số f đgl hàm số chẵn nếu với "x Î D thì –x Î D và f(–x) = f(x). · Hàm số f đgl hàm số lẻ nếu với "x Î D thì –x Î D và f(–x) = –f(x). Chú ý: + Đồ thị của hàm số chẵn nhận trục tung làm trục đối xứng. + Đồ thị của hàm số lẻ nhận gốc toạ độ làm tâm đối xứng. VẤN ĐỀ 1: Tìm tập xác định của hàm số · Đối với hàm số được cho bởi công thức y = f(x), tìm tập xác định D của hàm số y = f(x) là tìm tất cả những giá trị của biến số x sao cho biểu thức f(x) có nghĩa: D = { x Î R f ( x ) coù nghóa} . · Điều kiện xác định của một số hàm số thường gặp: P( x ) 1) Hàm số y = : Điều kiện xác định: Q(x) ¹ 0. Q( x ) 2) Hàm số y = R( x ) : Điều kiện xác định: R(x) ³ 0. Chú ý: + Đôi khi ta sử dụng phối hợp các điều kiện với nhau. + Điều kiện để hàm số xác định trên tập A là A Ì D. ìA ¹ 0 + A.B ¹ 0 Û í . îB ¹ 0 Trang 7 Hàm số bậc nhất – bậc hai Nguy n Công Nh t Baøi 1. Tính giá trị của các hàm số sau tại các điểm đã chỉ ra: a) f ( x ) = -5 x . Tính f(0), f(2), f(–2), f(3). b) f ( x ) = x -1 . Tính f(2), f(0), f(3), f(–2). 2 x - 3x + 1 c) f ( x ) = 2 x - 1 + 3 x - 2 . Tính f(2), f(–2), f(0), f(1). 2 ì 2 ïï x - 1 khi x < 0 d) f ( x ) = íï x + 1 khi 0 £ x £ 2 . Tính f(–2), f(0), f(1), f(2) f(3). ïî x 2 - 1 khi x > 2 ì-1 khi x < 0 ï e) f ( x ) = í0 khi x = 0 . Tính f(–2), f(–1), f(0), f(2), f(5). ïî1 khi x > 0 Baøi 2. Tìm tập xác định của các hàm số sau: 2x + 1 x -3 4 a) y = b) y = c) y = 3x + 2 5 - 2x x+4 x x -1 3x d) y = e) y = f) y = 2 2 2 x - 3x + 2 2 x - 5x + 2 x + x +1 x -1 2x +1 1 g) y = h) y = i) y = 3 2 4 x +1 ( x - 2)( x - 4 x + 3) x + 2 x2 - 3 ĐS: Baøi 3. Tìm tập xác định của các hàm số sau: a) y = 2 x - 3 d) y = x - 1 + g) y = b) y = 1 x -3 e) y = 5 - 2x 2x - 3 1 f) y = x + 3 - 2 x + 2 ( x + 2) x - 1 h) y = 2 x - 1 + ( x - 2) x - 1 c) y = 4 - x + x + 1 1 3- x i) y = x + 3 + ĐS: Baøi 4. Tìm a để hàm số xác định trên tập K đã chỉ ra: a) y = 2x + 1 2 x - 6x + a - 2 3x + 1 b) y = ; x 2 - 2ax + 4 K = R. ĐS: a > 11 K = R. ĐS: –2 < a < 2 K = (0; +¥). ĐS: a £ -1 x-a ; K = (0; +¥). x + a -1 ĐS: 1 £ a £ ; c) y = x - a + 2 x - a - 1 ; d) y = 2 x - 3a + 4 + x + 2a ; x - a +1 1 f) y = + - x + 2a + 6 ; x -a 1 e) y = 2 x + a + 1 + ; x-a e) y = 4 3 K = (–1; 0). ĐS: a £ 0 hoặc a ³ 1 K = (–1; 0). ĐS: –3 £ a £ –1 K = (1; +¥). ĐS: –1 £ a £ 1 Trang 8 1 2 x -4 Hàm số bậc nhất – bậc hai VẤN ĐỀ 2: Xét sự biến thiên của hàm số Cho hàm số f xác định trên K. · y = f(x) đồng biến trên K Û "x1 , x2 Î K : x1 < x2 Þ f ( x1 ) < f ( x2 ) Û "x1 , x2 Î K : x1 ¹ x2 Þ f ( x2 ) - f ( x1 ) >0 x2 - x1 · y = f(x) nghịch biến trên K Û "x1 , x2 Î K : x1 < x2 Þ f ( x1 ) > f ( x2 ) Û "x1 , x2 Î K : x1 ¹ x2 Þ f ( x2 ) - f ( x1 ) -1 ; NB khi m < -1 d) ĐB khi m < -1 ; NB khi m > -1 m x -2 ĐS: a) ĐB khi m > 2 ; NB khi m < 2 c) ĐB khi m < 0 ; NB khi m > 0 c) y = d) y = VẤN ĐỀ 3: Xét tính chẵn lẻ của hàm số Để xét tính chẵn lẻ của hàm số y = f(x) ta tiến hành các bước như sau: · Tìm tập xác định D của hàm số và xét xem D có là tập đối xứng hay không. · Nếu D là tập đối xứng thì so sánh f(–x) với f(x) (x bất kì thuộc D). + Nếu f(–x) = f(x), "x Î D thì f là hàm số chẵn. + Nếu f(–x) = –f(x), "x Î D thì f là hàm số lẻ. Chú ý: + D là tập đối xứng nếu thoả mãn điều kiện: Với "x Î D thì –x Î D. + Nếu $x Î D mà f(–x) ¹ ± f(x) thì f là hàm số không chẵn không lẻ. Baøi 1. Xét tính chẵn lẻ của các hàm số sau: a) y = x 4 - 4 x 2 + 2 d) y = x2 + 4 4 x ĐS: Chẵn: a, d b) y = -2 x 3 + 3 x e) y = x 3 + 1 x Lẻ: b, e Trang 9 c) y = ( x - 1)2 f) y = x 2 + x Hàm số bậc nhất – bậc hai Nguy n Công Nh t Baøi 2. Xét tính chẵn lẻ của các hàm số sau: a) y = x + 2 - x - 2 c) y = 2 x 2 - x b) y = 2 x + 1 + 2 x - 1 x +1 + x -1 e) y = 2 x + 1 x +1 - x -1 ĐS: Chẵn: b, c Lẻ: a, d Baøi 3. Xét tính chẵn lẻ của các hàm số sau: d) y = a) y = x + 2 + 2 - x b) y = x + 2 - 2 - x d) y = x + 3 + x - 3 e) y = x + 2 - 2 - x + ĐS: Chẵn: Lẻ: Trang 10 c) y = x + 3 + - x + 3 3 x Hàm số bậc nhất – bậc hai II. HÀM SỐ BẬC NHẤT 1. Hàm số bậc nhất y = ax + b (a ¹ 0) · Tập xác định: D = R. · Sự biến thiên: + Khi a > 0, hàm số đồng biến trên R. + Khi a < 0, hàm số nghịch biến trên R. · Đồ thị là đường thẳng có hệ số góc bằng a, cắt trục tung tại điểm B(0; b). Chú ý: Cho hai đường thẳng (d): y = ax + b và (d¢): y = a¢x + b¢: + (d) song song với (d¢) Û a = a¢ và b ¹ b¢. + (d) trùng với (d¢) Û a = a¢ và b = b¢. + (d) cắt (d¢) Û a ¹ a¢. 2. Hàm số y = ax + b (a ¹ 0) ì b khi x ³ ï ax + b a y = ax + b = í b ï-(ax + b) khi x < ïî a Chú ý: Để vẽ đồ thị của hàm số y = ax + b ta có thể vẽ hai đường thẳng y = ax + b và y = –ax – b, rồi xoá đi hai phần đường thẳng nằm ở phía dưới trục hoành. Baøi 1. Vẽ đồ thị của các hàm số sau: a) y = 2 x - 7 b) y = -3 x + 5 c) y = Baøi 2. Tìm toạ độ giao điểm của các cặp đường thẳng sau: x -3 2 a) y = 3x - 2; y = 2x + 3 b) y = -3 x + 2; c) y = 2 x; y = -x - 3 d) y = x -3 ; 2 d) y = 5- x 3 y = 4( x - 3) y= 5- x 3 ĐS: Baøi 3. Trong mỗi trường hợp sau, tìm giá trị k để đồ thị của hàm số y = -2 x + k ( x + 1) : a) Đi qua gốc tọa độ O b) Đi qua điểm M(–2; 3) c) Song song với đường thẳng y = 2.x ĐS: Baøi 4. Xác định a và b để đồ thị của hàm số y = ax + b : a) Đi qua hai điểm A(–1; –20), B(3; 8). 2 b) Đi qua điểm M(4; –3) và song song với đường thẳng d: y = - x + 1 . 3 c) Cắt đường thẳng d1: y = 2 x + 5 tại điểm có hoành độ bằng –2 và cắt đường thẳng d2: y = –3 x + 4 tại điểm có tung độ bằng –2. d) Song song với đường thẳng y = 1 x và đi qua giao điểm của hai đường thẳng 2 1 y = - x + 1 và y = 3 x + 5 . 2 ĐS: Baøi 5. Trong mỗi trường hợp sau, tìm các giá trị của m sao cho ba đường thẳng sau phân biệt và đồng qui: a) y = 2 x; y = - x - 3; y = mx + 5 Trang 11 Hàm số bậc nhất – bậc hai Nguy n Công Nh t b) y = –5( x + 1); y = mx + 3; y = 3x + m c) y = 2 x - 1; y = 8 - x; y = (3 - 2m) x + 2 d) y = (5 - 3m) x + m - 2; y = - x + 11; y = x + 3 e) y = - x + 5; y = 2 x - 7; y = (m - 2) x + m2 + 4 ĐS: Baøi 6. Tìm điểm sao cho đường thẳng sau luôn đi qua dù m lấy bất cứ giá trị nào: a) y = 2mx + 1 - m b) y = mx - 3 - x c) y = (2m + 5) x + m + 3 d) y = m( x + 2) e) y = (2m - 3) x + 2 f) y = (m - 1) x - 2m ĐS: Baøi 7. Với giá trị nào của m thì hàm số sau đồng biến? nghịch biến? a) y = (2m + 3) x - m + 1 b) y = (2m + 5) x + m + 3 c) y = mx - 3 - x d) y = m( x + 2) ĐS: Baøi 8. Tìm các cặp đường thẳng song song trong các đường thẳng cho sau đây: x a) 3y - 6 x + 1 = 0 b) y = -0,5 x - 4 c) y = 3 + d) 2 y + x = 6 2 e) 2 x - y = 1 f) y = 0,5 x + 1 ĐS: Baøi 9. Với giá trị nào của m thì đồ thị của các cặp hàm số sau song song với nhau: m 2(m + 2) 3m 5m + 4 a) y = (3m - 1) x + m + 3; y = 2 x - 1 b) y = x+ ; y= x1- m m -1 3m + 1 3m + 1 c) y = m( x + 2); y = (2m + 3) x - m + 1 ĐS: Baøi 10. Vẽ đồ thị của các hàm số sau: ì- x ì-2 x - 2 khi x £ -1 khi x < -1 ï ï a) y = í1 khi - 1 < x < 2 b) y = í0 khi - 1 £ x £ 2 ïî x - 1 khi x ³ 2 ïî x - 2 khi x ³ 2 1 5 c) y = 3x + 5 d) y = -2 x - 1 e) y = - 2 x + 3 + 2 2 f) y = x - 2 + 1 - x g) y = x - x - 1 h) y = x + x - 1 + x + 1 Trang 12 Hàm số bậc nhất – bậc hai III. HÀM SỐ BẬC HAI y = ax 2 + bx + c (a ¹ 0) · Tập xác định: D = R · Sự biến thiên: æ b D ö b · Đồ thị là một parabol có đỉnh I ç - ; - ÷ , nhận đường thẳng x = làm trục đối 2a è 2a 4a ø xứng, hướng bề lõm lên trên khi a > 0, xuống dưới khi a < 0. Chú ý: Để vẽ đường parabol ta có thể thực hiện các bước như sau: æ b D ö – Xác định toạ độ đỉnh I ç - ; - ÷ . è 2a 4a ø b – Xác định trục đối xứng x = và hướng bề lõm của parabol. 2a – Xác định một số điểm cụ thể của parabol (chẳng hạn, giao điểm của parabol với các trục toạ độ và các điểm đối xứng với chúng qua trục trục đối xứng). – Căn cứ vào tính đối xứng, bề lõm và hình dáng parabol để vẽ parabol. Baøi 1. Xét sự biến thiên và vẽ đồ thị của các hàm số sau: a) y = x 2 - 2 x b) y = - x 2 + 2 x + 3 c) y = - x 2 + 2 x - 2 1 d) y = - x 2 + 2 x - 2 e) y = x 2 - 4 x + 4 f) y = - x 2 - 4 x + 1 2 Baøi 2. Tìm toạ độ giao điểm của các cặp đồ thị của các hàm số sau: a) y = x - 1; y = x2 - 2x - 1 b) y = - x + 3; c) y = 2 x - 5; y = x2 - 4x + 4 d) y = x 2 - 2 x - 1; y = x 2 - 4 x + 4 e) y = 3x 2 - 4 x + 1; y = -3 x 2 + 2 x - 1 ĐS: y = -x2 - 4x + 1 f) y = 2 x 2 + x + 1; y = - x 2 + x - 1 Baøi 3. Xác định parabol (P) biết: a) (P): y = ax 2 + bx + 2 đi qua điểm A(1; 0) và có trục đối xứng x = 3 . 2 b) (P): y = ax 2 + bx + 3 đi qua điểm A(–1; 9) và có trục đối xứng x = -2 . c) (P): y = ax 2 + bx + c đi qua điểm A(0; 5) và có đỉnh I(3; –4). d) (P): y = ax 2 + bx + c đi qua điểm A(2; –3) và có đỉnh I(1; –4). e) (P): y = ax 2 + bx + c đi qua các điểm A(1; 1), B(–1; –3), O(0; 0). f) (P): y = x 2 + bx + c đi qua điểm A(1; 0) và đỉnh I có tung độ bằng –1. ĐS: Trang 13 Hàm số bậc nhất – bậc hai Nguy n Công Nh t Baøi 4. Chứng minh rằng với mọi m, đồ thị của mỗi hàm số sau luôn cắt trục hoành tại hai điểm phân biệt và đỉnh I của đồ thị luôn chạy trên một đường thẳng cố định: m2 a) y = x - mx + -1 4 ĐS: 2 b) y = x 2 - 2mx + m 2 - 1 Baøi 5. Vẽ đồ thị của hàm số y = - x 2 + 5 x + 6 . Hãy sử dụng đồ thị để biện luận theo tham số m, số điểm chung của parabol y = - x 2 + 5 x + 6 và đường thẳng y = m . Baøi 6. Vẽ đồ thị của các hàm số sau: a) y = x 2 - 2 x + 1 b) y = x ( x - 2 ) c) y = x 2 - 2 x - 1 ìï- x 2 - 2 ì-2 x + 1 ì2 x neáu x ³ 0 khi x < 0 neáu x < 1 d) y = í 2 e) y = í 2 f) y = í 2 ïî2 x - 2 x - 3 neáu x ³ 1 î x + 4 x + 1 neáu x < 0 î x - x khi x ³ 0 Trang 14 Hàm số bậc nhất – bậc hai BÀI TẬP ÔN CHƯƠNG II Bài 1. Tìm tập xác định của các hàm số sau: a) y = 2 - x d) y = 4 x+4 x2 + 2x + 3 2- 5- x ĐS: a) ( -4;2] b) y = 1- x - 1+ x x c) y = e) y = x + 2 + 3 - 2x x -1 f) y = é 3 ù d) ê - ;5ú \ {1} ë 2 û b) [ -1;1] \ {0} c) R \ {1} f) (2; +¥) Bài 2. Xét sự biến thiên của các hàm số sau: x +1 a) y = - x 2 + 4 x - 1 trên (-¥; 2) b) y = trên (1; +¥) x -1 d) y = 3 - 2 x e) y = 1 x -2 ĐS: a) ĐB b) NB c) NB Bài 3. Xét tính chẵn lẻ của các hàm số sau: a) y = x4 + x2 - 2 2 x -1 d) NB b) y = 3 + x + 3 - x c) y = 3x 2 - x x2 - x + x - 1 2x -1 x x -4 é 3ù e) ê -2; ú \ { - 1;1} ë 2û 1 x -1 x +3 f) y = trên (2; +∞) x -2 e) NB f) NB c) y = x ( x 2 + 2 x ) 3 x +1 + x -1 x x e) y = f) y = x - 2 x +1 - x -1 x2 + 1 ĐS: a) chẵn b) chẵn c) lẻ d) lẻ e) lẻ f) không chẵn, không lẻ Bài 4. Giả sử y = f(x) là hàm số xác định trên tập đối xứng D. Chứng minh rằng: 1 a) Hàm số F ( x ) = [ f ( x ) + f (- x )] là hàm số chẵn xác định trên D. 2 1 b) Hàm số G( x ) = [ f ( x ) - f (- x )] là hàm số lẻ xác định trên D. 2 c) Hàm số f(x) có thể phân tích thành tổng của một hàm số chẵn và một hàm số lẻ. d) y = Bài 5. Cho hàm số y = ax 2 + bx + c (P). · Tìm a, b, c thoả điều kiện được chỉ ra. · Khảo sát sự biến thiên và vẽ đồ thị (P) của hàm số vừa tìm được. · Tìm m để đường thẳng d cắt (P) tại hai điểm phân biệt A và B. Xác định toạ độ trung điểm I của đoạn AB. æ1 3ö a) (P) có đỉnh S ç ; ÷ và đi qua điểm A(1; 1); d: y = mx . è2 4ø b) (P) có đỉnh S(1; 1) và đi qua điểm A(0; 2); d: y = 2 x + m . ĐS: Trang 15 Phương trình bậc nhất – bậc hai Nguy n Công Nh t CHƯƠNG III PHƯƠNG TRÌNH VÀ HỆ PHƯƠNG TRÌNH I. ĐẠI CƯƠNG VỀ PHƯƠNG TRÌNH 1. Phương trình một ẩn f(x) = g(x) (1) · x0 là một nghiệm của (1) nếu "f(x0) = g(x0)" là một mệnh đề đúng. · Giải phương trình là tìm tất cả các nghiệm của phương trình đó. · Khi giải phương trình ta thường tìm điều kiện xác định của phương trình. Chú ý: + Khi tìm ĐKXĐ của phương trình, ta thường gặp các trường hợp sau: 1 – Nếu trong phương trình có chứa biểu thức thì cần điều kiện P(x) ¹ 0. P( x ) – Nếu trong phương trình có chứa biểu thức P( x ) thì cần điều kiện P(x) ³ 0. + Các nghiệm của phương trình f(x) = g(x) là hoành độ các giao điểm của đồ thị hai hàm số y = f(x) và y = g(x). 2. Phương trình tương đương, phương trình hệ quả Cho hai phương trình f1(x) = g1(x) (1) có tập nghiệm S1 và f2(x) = g2(x) (2) có tập nghiệm S2. · (1) Û (2) khi và chỉ khi S1 = S2. · (1) Þ (2) khi và chỉ khi S1 Ì S2. 3. Phép biến đổi tương đương · Nếu một phép biến đổi phương trình mà không làm thay đổi điều kiện xác định của nó thì ta được một phương trình tương đương. Ta thường sử dụng các phép biến đổi sau: – Cộng hai vế của phương trình với cùng một biểu thức. – Nhân hai vế của phương trình với một biểu thức có giá trị khác 0. · Khi bình phương hai vế của một phương trình, nói chung ta được một phương trình hệ quả. Khi đó ta phải kiểm tra lại để loại bỏ nghiệm ngoại lai. Tìm điều kiện xác định của mỗi phương trình và giải phương trình đó: 5 5 1 1 a) 3 x + = 12 + b) 5 x + = 15 + x-4 x -4 x+3 x +3 1 1 2 2 c) x 2 = 9d) 3 x + = 15 + x -1 x -1 x -5 x-5 ĐS: Bài 2. Tìm điều kiện xác định của mỗi phương trình và giải phương trình đó: Bài 1. a) 1 + 1 - x = x - 2 c) e) x +1 = x +1 x 3 = x -1 x -1 b) x +1 = 2 - x d) x -1 = 1- x f) x 2 - 1 - x = x - 2 + 3 ĐS: Bài 3. a) Tìm điều kiện xác định của mỗi phương trình và giải phương trình đó: x - 3( x 2 - 3x + 2) = 0 b) x + 1( x 2 - x - 2) = 0 Trang 16 Phương trình bậc nhất – bậc hai c) x x -2 = 1 x-2 - x-2 d) x2 - 4 x +1 = x +3 x +1 + x +1 ĐS: Tìm điều kiện xác định của mỗi phương trình và giải phương trình đó: a) x - 2 = x + 1 b) x + 1 = x - 2 c) 2 x - 1 = x + 2 d) x - 2 = 2 x - 1 ĐS: Bài 5. Tìm điều kiện xác định của mỗi phương trình và giải phương trình đó: x x x-2 x -2 a) = b) = x -1 x -1 x -1 x -1 x x x -1 1- x c) = d) = 2- x 2-x x -2 x-2 ĐS: Bài 4. Bài 6. a) Trang 17 Phương trình bậc nhất – bậc hai Nguy n Công Nh t II. PHƯƠNG TRÌNH ax + b = 0 ax + b = 0 Hệ số (1) có nghiệm duy nhất x = - a¹0 a=0 (1) Kết luận b¹0 b=0 b a (1) vô nghiệm (1) nghiệm đúng với mọi x Chú ý: Khi a ¹ 0 thì (1) đgl phương trình bậc nhất một ẩn. Bài 1. Giải và biện luận các phương trình sau theo tham số m: a) (m 2 + 2) x - 2m = x - 3 b) m( x - m) = x + m - 2 c) m( x - m + 3) = m( x - 2) + 6 d) m 2 ( x - 1) + m = x (3m - 2) e) (m 2 - m) x = 2 x + m2 - 1 ĐS: f) (m + 1)2 x = (2m + 5) x + 2 + m Bài 2. Giải và biện luận các phương trình sau theo các tham số a, b, c: a) x-a x -b -b = - a (a, b ¹ 0) a b b) (ab + 2) x + a = 2b + (b + 2a) x x + ab x + bc x + b2 + + = 3b (a, b, c ¹ -1) a +1 c +1 b +1 x -b-c x -c-a x -a-b d) + + = 3 (a, b, c ¹ 0) a b c ĐS: c) Bài 3. Trong các phương trình sau, tìm giá trị của tham số để phương trình: i) Có nghiệm duy nhất ii) Vô nghiệm iii) Nghiệm đúng với mọi x Î R. a) (m - 2) x = n - 1 b) (m 2 + 2m - 3) x = m - 1 c) (mx + 2)( x + 1) = (mx + m 2 ) x ĐS: Nghiệm a) b) Duy nhất Vô nghiệm "x Î R d) (m 2 - m) x = 2 x + m2 - 1 c) Trang 18 d) e) f) Phương trình bậc nhất – bậc hai III. PHƯƠNG TRÌNH BẬC HAI ax2 + bx + c = 0 (a ¹ 0) 1. Cách giải ax2 + bx + c = 0 D = b2 - 4ac Chú ý: (a ¹ 0) (1) Kết luận D>0 (1) có 2 nghiệm phân biệt x1,2 = D=0 (1) có nghiệm kép x = - D 0 ìD ³ 0 ìD ³ 0 ï ï · (1) có hai nghiệm dương Û í P > 0 · (1) có hai nghiệm âm Û í P > 0 ïîS > 0 ïîS < 0 Chú ý: Trong các trường hợp trên nếu yêu cầu hai nghiệm phân biệt thì D > 0. · (1) có hai nghiệm trái dấu Û P < 0 Bài 1. Xác định m để phương trình: i) có hai nghiệm trái dấu iii) có hai nghiệm dương phân biệt ii) có hai nghiệm âm phân biệt a) x 2 + 5 x + 3m - 1 = 0 b) 2 x 2 + 12 x - 15m = 0 c) x 2 - 2(m - 1) x + m 2 = 0 d) (m + 1) x 2 - 2(m - 1) x + m - 2 = 0 e) (m - 1) x 2 + (2 - m) x - 1 = 0 f) mx 2 - 2(m + 3) x + m + 1 = 0 g) x 2 - 4 x + m + 1 = 0 ĐS: h) (m + 1) x 2 + 2(m + 4) x + m + 1 = 0 VẤN ĐỀ 3: Một số bài tập áp dụng định lí Vi–et 1. Biểu thức đối xứng của các nghiệm số b c Ta sử dụng công thức S = x1 + x2 = - ; P = x1 x2 = để biểu diễn các biểu thức đối a a xứng của các nghiệm x1, x2 theo S và P. Ví dụ: x12 + x22 = ( x1 + x2 )2 - 2 x1 x2 = S 2 - 2 P x13 + x23 = ( x1 + x2 ) éë( x1 + x2 )2 - 3 x1 x2 ùû = S(S 2 - 3P ) 2. Hệ thức của các nghiệm độc lập đối với tham số Để tìm hệ thức của các nghiệm độc lập đối với tham số ta tìm: b c S = x1 + x2 = - ; P = x1 x2 = (S, P có chứa tham số m). a a Khử tham số m giữa S và P ta tìm được hệ thức giữa x1 và x2. 3. Lập phương trình bậc hai Nếu phương trình bậc hai có các nghiệm u và v thì phương trình bậc hai có dạng: x 2 - Sx + P = 0 , trong đó S = u + v, P = uv. Bài 1. Gọi x1, x2 là các nghiệm của phương trình. Không giải phương trình, hãy tính: A = x12 + x22 ; B = x13 + x23 ; C = x14 + x24 ; D = x1 - x2 ; E = (2 x1 + x2 )(2 x2 + x1 ) a) x 2 - x - 5 = 0 b) 2 x 2 - 3 x - 7 = 0 c) 3 x 2 + 10 x + 3 = 0 d) x 2 - 2 x - 15 = 0 ĐS: e) 2 x 2 - 5 x + 2 = 0 f) Trang 20 3x 2 + 5x - 2 = 0 Phương trình bậc nhất – bậc hai Câu A B C D a) b) c) d) e) f) Bài 2. Cho phương trình: (m + 1) x 2 - 2(m - 1) x + m - 2 = 0 (*). Xác định m để: a) (*) có hai nghiệm phân biệt. b) (*) có một nghiệm bằng 2. Tính nghiệm kia. c) Tổng bình phương các nghiệm bằng 2. ĐS: Bài 3. Cho phương trình: x 2 - 2(2m + 1) x + 3 + 4m = 0 (*). a) Tìm m để (*) có hai nghiệm x1, x2. b) Tìm hệ thức giữa x1, x2 độc lập đối với m. c) Tính theo m, biểu thức A = x13 + x23 . d) Tìm m để (*) có một nghiệm gấp 3 lần nghiệm kia. e) Lập phương trình bậc hai có các nghiệm là x12 , x22 . HD: a) m ³ d) m = 2 2 1± 2 7 6 b) x1 + x2 - x1 x2 = -1 c) A = (2 + 4m)(16m2 + 4m - 5) e) x 2 - 2(8m 2 + 4m - 1) x + (3 + 4m)2 = 0 Bài 4. Cho phương trình: x 2 - 2(m - 1) x + m 2 - 3m = 0 (*). a) Tìm m để (*) có nghiệm x = 0. Tính nghiệm còn lại. b) Khi (*) có hai nghiệm x1, x2 . Tìm hệ thức giữa x1, x2 độc lập đối với m. c) Tìm m để (*) có hai nghiệm x1, x2 thoả: x12 + x22 = 8 . HD: a) m = 3; m = 0 b) ( x1 + x2 )2 - 2( x1 + x2 ) - 4 x1 x2 - 8 = 0 c) m = –1; m = 2. Bài 5. Cho phương trình: x 2 + (m 2 - 3m) x + m3 = 0 . a) Tìm m để phương trình có một nghiệm bằng bình phương nghiệm kia. b) Tìm m để phương trình có một nghiệm bằng 1. Tính nghiệm còn lại. HD: a) m = 0; m = 1 b) x2 = 1; x2 = 5 2 - 7; x2 = -5 2 - 7 . Bài 6. (nâng cao) Cho phương trình: 2 x 2 + 2 x sin a = 2 x + cos2 a (a là tham số). a) Chứng minh phương trình có nghiệm với mọi a. b) Tìm a để tổng bình phương các nghiệm của phương trình đạt GTLN, GTNN. ĐS: Trang 21 Phương trình bậc nhất – bậc hai Nguy n Công Nh t IV. PHƯƠNG TRÌNH CHỨA ẨN TRONG DẤU GIÁ TRỊ TUYỆT ĐỐI 1. Định nghĩa và tính chất ìA khi A ³ 0 · A =í A khi A < 0 î · A ³ 0, "A 2 · A.B = A . B · A = A2 · A + B = A + B Û A.B ³ 0 · A - B = A + B Û A.B £ 0 · A + B = A - B Û A.B £ 0 · A - B = A - B Û A.B ³ 0 2. Cách giải Để giải phương trình chứa ẩn trong dấu GTTĐ ta tìm cách để khử dấu GTTĐ, bằng cách: – Dùng định nghĩa hoặc tính chất của GTTĐ. – Bình phương hai vế. – Đặt ẩn phụ. éì f ( x) ³ 0 C1 ê í C2 ì g( x ) ³ 0 f ( x ) = g( x ) ï î · Dạng 1: f ( x ) = g( x ) Û ê Û í é f ( x ) = g( x ) ê ìí f ( x ) < 0 ïî êë f ( x ) = - g( x ) êë î- f ( x ) = g( x ) · Dạng 2: C1 2 2 f ( x ) = g( x ) Û [ f ( x )] = [ g( x )] C2 é f ( x ) = g( x ) Ûê ë f ( x ) = - g( x ) · Dạng 3: a f ( x ) + b g( x ) = h( x ) Đối với phương trình có dạng này ta thường dùng phương pháp khoảng để giải. Bài 1. Giải các phương trình sau: c) x 2 - 3 x + 2 = 0 a) 2 x - 1 = x + 3 b) 4 x + 7 = 2 x + 5 d) x 2 + 6 x + 9 = 2 x - 1 g) x - 1 - x + 2 x + 3 = 2 x + 4 e) x 2 - 4 x - 5 = 4 x - 17 f) 4 x - 17 = x 2 - 4 x - 5 h) x - 1 + x + 2 + x - 3 = 14 i) x - 1 + 2 - x = 2 x 2 c) x = ±1; x = ±2 d) x = 4; x = e) 3 h) i) ĐS: a) b) f) g) Bài 2. Giải các phương trình sau: a) 4 x + 7 = 4 x + 7 d) x 2 - 2 x - 3 = x 2 + 2 x + 3 7 3 ĐS: a) x ³ b) x £ 4 2 f) -3 £ x £ 7 Bài 3. Giải các phương trình sau: a) x 2 - 2 x + x - 1 - 1 = 0 b) 2 x - 3 = 3 - 2 x c) x - 1 + 2 x + 1 = 3 x e) 2 x - 5 + 2 x 2 - 7 x + 5 = 0 f) x + 3 + 7 - x = 10 1 3 5 c) x £ - Ú x ³ 1 d) x £ - Ú x = 0 e) x = 2 2 2 b) x 2 - 2 x - 5 x - 1 + 7 = 0 c) x 2 - 2 x - 5 x - 1 - 5 = 0 d) x 2 + 4 x + 3 x + 2 = 0 e) 4 x 2 - 4 x - 2 x - 1 - 1 = 0 ĐS: a) x = 0; x = 2 b) c) d) Bài 4. Giải và biện luận các phương trình sau: a) mx - 1 = 5 b) mx - x + 1 = x + 2 f) x 2 + 6 x + x + 3 + 10 = 0 e) f) Trang 22 c) mx + 2 x - 1 = x Phương trình bậc nhất – bậc hai d) 3 x + m = 2 x - 2m ĐS: e) x + m = x - m + 2 f) x - m = x + 1 Bài 5. Tìm các giá trị của tham số m sao cho phương trình sau có nghiệm duy nhất: a) mx - 2 = x + 4 ĐS: b) mx - x + 1 = x + 2 Trang 23 c) 3 x + m = 2 x - 2m Phương trình bậc nhất – bậc hai Nguy n Công Nh t V. PHƯƠNG TRÌNH CHỨA ẨN DƯỚI DẤU CĂN Cách giải: Để giải phương trình chứa ẩn dưới dấu căn ta tìm cách để khử dấu căn, bằng cách: – Nâng luỹ thừa hai vế. – Đặt ẩn phụ. Chú ý: – Khi thực hiện các phép biến đổi cần chú ý điều kiện để các căn thức được xác định. – Muốn bình phương 2 vế của một phương trình thì cả 2 vế đều phải không âm. Một số phương pháp giải 1. Nâng luỹ thừa hai vế Dạng 1: Dạng 2: 2 ìï f ( x ) = g( x ) Û í f ( x ) = [ g( x )] ïî g( x ) ³ 0 ì f ( x ) = g( x ) f ( x ) = g( x ) Û í î f ( x ) ³ 0 (hay g( x ) ³ 0) 2. Đặt ẩn phụ Dạng 3: Dạng 4: ìït = f ( x ), t ³ 0 af ( x ) + b f ( x ) + c = 0 Û í 2 ïîat + bt + c = 0 f ( x ) + g( x ) = h( x ) · Đặt u = f ( x ), v = g( x ) với u, v ³ 0. · Đưa phương trình trên về hệ phương trình với hai ẩn là u và v. Dạng 5: f ( x ) + g( x ) + f ( x ).g( x ) = h( x ) Đặt t = f ( x ) + g( x ), t ³ 0 . 3. Phương pháp đối lập ìA ³ M ï ìA = M Nếu ta có í B £ M thì í îB = M ïî A = B Bài 1. Giải các phương trình sau: a) 2x - 3 = x - 3 b) 5 x + 10 = 8 - x c) x - 2 x - 5 = 4 d) x 2 + x - 12 = 8 - x e) x2 + 2 x + 4 = 2 - x f) 3 x 2 - 9 x + 1 = x - 2 h) c) h) x 2 - 3 x - 10 = x - 2 d) i) i) ( x - 3) x 2 + 4 = x 2 - 9 e) x = -1; x = -2 g) 3x 2 - 9 x + 1 = x - 2 ĐS: a) x = 6 b) f) g) Bài 2. Giải các phương trình sau: a) x 2 - 6 x + 9 = 4 x 2 - 6 x + 6 b) c) ( x + 4)( x + 1) - 3 x 2 + 5 x + 2 = 6 d) ( x + 5)(2 - x ) = 3 x 2 + 3 x e) x 2 + x 2 + 11 = 31 ĐS: a) b) Bài 3. Giải các phương trình sau: a) x +1 - x -1 = 1 ( x - 3)(8 - x ) + 26 = - x 2 + 11x f) x 2 - 2 x + 8 - 4 (4 - x )( x + 2) = 0 c) x = 2; x = -7 d) e) f) b) 3x + 7 - x + 1 = 2 Trang 24 Phương trình bậc nhất – bậc hai x2 + 9 - x2 - 7 = 2 d) 3x 2 + 5 x + 8 - 3 x 2 + 5 x + 1 = 1 e) 3 1 + x + 3 1 - x = 2 f) x 2 + x - 5 + x 2 + 8x - 4 = 5 c) 5 x + 7 - 3 5 x - 13 = 1 5 ĐS: a) x = b) 4 g) 3 h) 3 9 - x +1 + 3 7 + x +1 = 4 Bài 4. Giải các phương trình sau: a) x + 3 + 6 - x = 3 + ( x + 3)(6 - x ) b) 2 x + 3 + x + 1 = 3 x + 2 (2 x + 3)( x + 1) - 16 c) x - 1 + 3 - x - ( x - 1)(3 - x ) = 1 7 - x + 2 + x - (7 - x )(2 + x ) = 3 e) x + 1 + 4 - x + ( x + 1)(4 - x ) = 5 f) g) 1 + ĐS: 2 x - x2 = x + 1 - x 3 d) h) 3x - 2 + x - 1 = 4 x - 9 + 2 3 x 2 - 5 x + 2 x + 9 - x = - x2 + 9x + 9 Bài 5. Giải các phương trình sau: a) 2 x - 4 + 2 2 x - 5 + 2 x + 4 + 6 2 x - 5 = 14 b) x + 5 - 4 x +1 + x + 2 - 2 x +1 = 1 c) 2 x - 2 2 x - 1 - 2 2 x + 3 - 4 2 x - 1 + 3 2 x + 8 - 6 2 x - 1 = 4 ĐS: Bài 6. Giải các phương trình sau: (phương pháp đối lập) a) 3 x 2 + 6 x + 7 + 5 x 2 + 10 x + 14 = 4 - 2 x - x 2 b) c) 3 x - 5 + 7 - 3 x = 5 x 2 - 20 x + 22 ĐS: d) Trang 25 x - 2 + 10 - x = x 2 - 12 x + 40 x2 - 4 x + 4 + x2 - 6 x + 9 = 1 Phương trình bậc nhất – bậc hai Nguy n Công Nh t VI. PHƯƠNG TRÌNH CHỨA ẨN Ở MẪU THỨC Cách giải: Khi giải phương trình chứa ẩn ở mẫu thức, ta phải chú ý đến điều kiện xác định của phương trình (mẫu thức khác 0). Bài 1. Giải các phương trình sau: a) 1 + c) 2 10 50 = x - 2 x + 3 (2 - x )( x + 3) 2x + 1 x + 1 = 3x + 2 x - 2 b) x + 1 x -1 2x + 1 + = x + 2 x - 2 x +1 x2 - 3x + 5 = -1 x2 - 4 x +3 4x - 2 f) = 2 ( x + 1) (2 x - 1)2 d) 2 x 2 - 5 x + 2 2 x 2 + x + 15 = x -1 x -3 ĐS: e) Bài 2. Giải và biện luận các phương trình sau: mx - m + 1 =3 x+2 x+m x+3 d) = x -1 x - 2 ĐS: a) mx + m - 2 =3 x-m (m + 1) x + m - 2 e) =m x +3 b) Trang 26 x - m x -1 + =2 x -1 x - m x x f) = x+m x +1 c) Phương trình bậc nhất – bậc hai VII. PHƯƠNG TRÌNH TRÙNG PHƯƠNG ax4 + bx2 + c = 0 (a ¹ 0) ìït = x 2 , t ³ 0 1. Cách giải: ax 4 + bx 2 + c = 0 (1) Û í 2 ïîat + bt + c = 0 (2) 2. Số nghiệm của phương trình trùng phương Để xác định số nghiệm của (1) ta dựa vào số nghiệm của (2) và dấu của chúng. é(2) voâ nghieäm · (1) vô nghiệm Û ê(2) coù nghieäm keùp aâm ê ë(2) coù 2 nghieäm aâm é(2) coù nghieäm keùp baèng 0 · (1) có 1 nghiệm Û ê ë(2) coù 1 nghieäm baèng 0, nghieäm coøn laïi aâm é(2) coù nghieäm keùp döông · (1) có 2 nghiệm Û ê ë(2) coù 1 nghieäm döông vaø 1 nghieäm aâm · (1) có 3 nghiệm Û (2) coù 1 nghieäm baèng 0, nghieäm coøn laïi döông · (1) có 4 nghiệm Û (2) coù 2 nghieäm döông phaân bieät 3. Một số dạng khác về phương trình bậc bốn · Dạng 1: ( x + a)( x + b)( x + c)( x + d ) = K , vôùi a + b = c + d – Đặt t = ( x + a)( x + b) Þ ( x + c)( x + d ) = t - ab + cd t 2 + (cd - ab)t - K = 0 – PT trở thành: · Dạng 2: ( x + a)4 + ( x + b)4 = K a+b a-b b-a Þ x+a=t+ , x+b=t+ 2 2 2 æ a-bö – PT trở thành: 2t 4 + 12a 2t 2 + 2a 4 - K = 0 ç vôùi a = ÷ è 2 ø – Đặt t = x + · Dạng 3: ax 4 + bx 3 + cx 2 ± bx + a = 0 (a ¹ 0) (phương trình đối xứng) – Vì x = 0 không là nghiệm nên chia hai vế của phương trình cho x 2 , ta được: æ æ 1 ö 1ö PT Û a ç x 2 + ÷ + b ç x ± ÷ + c = 0 (2) 2 xø è x ø è – Đặt t = x + 1æ 1ö ç hoaëc t = x - ÷ với t ³ 2 . xè xø – PT (2) trở thành: at 2 + bt + c - 2a = 0 ( t ³ 2) . Bài 1. Giải các phương trình sau: a) x 4 - 3x 2 - 4 = 0 b) x 4 - 5 x 2 + 4 = 0 c) x 4 + 5 x 2 + 6 = 0 d) 3 x 4 + 5 x 2 - 2 = 0 ĐS: e) x 4 + x 2 - 30 = 0 f) x 4 + 7 x 2 - 8 = 0 ii) Có 1 nghiệm v) Có 4 nghiệm iii) Có 2 nghiệm Bài 2. Tìm m để phương trình: i) Vô nghiệm iv) Có 3 nghiệm Trang 27 Phương trình bậc nhất – bậc hai Nguy n Công Nh t a) x 4 + (1 - 2m) x 2 + m 2 - 1 = 0 c) x 4 + 8mx 2 - 16m = 0 ĐS: Vô nghiệm a) b) c) b) x 4 - (3m + 4) x 2 + m 2 = 0 1 nghiệm 2 nghiệm Bài 3. Giải các phương trình sau: 3 nghiệm 4 nghiệm a) ( x - 1)( x - 3)( x + 5)( x + 7) = 297 b) ( x + 2)( x - 3)( x + 1)( x + 6) = -36 c) x 4 + ( x - 1)4 = 97 d) ( x + 4)4 + ( x + 6)4 = 2 e) ( x + 3)4 + ( x + 5)4 = 16 f) 6 x 4 - 35 x 3 + 62 x 2 - 35 x + 6 = 0 g) x 4 + x 3 - 4 x 2 + x + 1 = 0 ĐS: Trang 28 Phương trình bậc nhất – bậc hai VIII. HỆ PHƯƠNG TRÌNH BẬC NHẤT NHIỀU ẨN 1. Hệ phương trình bậc nhất hai ẩn ìa1x + b1y = c1 ía x + b y = c î 2 2 2 (a12 + b12 ¹ 0, a22 + b22 ¹ 0) Giải và biện luận: – Tính các định thức: D = a1 b1 a2 b2 Xét D c1 b1 c2 b2 , Dy = a1 c1 a2 c2 . Kết quả æ Dy ö D Hệ có nghiệm duy nhất ç x = x ; y = ÷ è D D ø Hệ vô nghiệm Hệ có vô số nghiệm D¹0 D=0 , Dx = Dx ¹ 0 hoặc Dy ¹ 0 Dx = Dy = 0 Chú ý: Để giải hệ phương trình bậc nhất hai ẩn ta có thể dùng các cách giải đã biết như: phương pháp thế, phương pháp cộng đại số. 2. Hệ phương trình bậc nhất nhiều ẩn Nguyên tắc chung để giải các hệ phương trình nhiều ẩn là khử bớt ẩn để đưa về các phương trình hay hệ phương trình có số ẩn ít hơn. Để khử bớt ẩn, ta cũng có thể dùng các phương pháp cộng đại số, phương pháp thế như đối với hệ phương trình bậc nhất hai ẩn. Giải các hệ phương trình sau: ì5 x - 4 y = 3 a) í b) î7 x - 9 y = 8 ì3 x - y = 1 c) í î6 x - 2 y = 5 ìï( 2 + 1) x + y = 2 - 1 d) í ïî2 x - ( 2 - 1) y = 2 2 ìï 3 x - y = 1 f) í ïî5x + 2 y = 3 Bài 1. ì2 x + y = 11 í5 x - 4 y = 8 î ì3 2 ï 4 x + 3 y = 16 e) í ï 5 x - 3 y = 11 î2 5 ĐS: Giải các hệ phương trình sau: ì1 8 ïï - = 18 a) í x y b) 5 4 ï + = 51 îï x y Bài 2. ì 10 1 ïï x - 1 + y + 2 = 1 í ï 25 + 3 = 2 îï x - 1 y + 2 ì2 x - 6 + 3 y + 1 = 5 ì2 x + y - x - y = 9 d) í e) í î5 x - 6 - 4 y + 1 = 1 î3 x + y + 2 x - y = 17 æ 11 44 ö ĐS: a) ç ;- ÷ b) è 120 39 ø Bài 3. Giải và biện luận các hệ phương trình sau: ìmx + (m - 1)y = m + 1 ì mx + (m - 2) y = 5 a) í b) í 2 x + my = 2 î î(m + 2) x + (m + 1) y = 2 ì (m + 4) x - (m + 2) y = 4 d) í î(2m - 1) x + (m - 4) y = m ĐS: ì 27 32 ïï 2 x - y + x + 3y = 7 c) í ï 45 - 48 = -1 îï 2 x - y x + 3y ì4 x + y + 3 x - y = 8 f) í î3 x + y - 5 x - y = 6 ì(m - 1) x + 2 y = 3m - 1 c) í î (m + 2) x - y = 1 - m ì(m + 1) x - 2 y = m - 1 ì mx + 2 y = m + 1 e) í f) í 2 2 m x - y = m + 2m î2 x + my = 2m + 5 î Trang 29 Phương trình bậc nhất – bậc hai Nguy n Công Nh t Trong các hệ phương trình sau hãy: i) Giải và biện luận. ii) Tìm m Î Z để hệ có nghiệm duy nhất là nghiệm nguyên. ì(m + 1) x - 2 y = m - 1 ì ìmx + y - 3 = 3 mx - y = 1 a) í b) í c) í 2 2 x + 4( m + 1) y = 4 m m x - y = m + 2m î î x + my - 2m + 1 = 0 î ĐS: Bài 5. Trong các hệ phương trình sau hãy: i) Giải và biện luận. ii) Khi hệ có nghiệm (x; y), tìm hệ thức giữa x, y độc lập đối với m. ì mx + 2 y = m + 1 ì6mx + (2 - m) y = 3 ìmx + (m - 1)y = m + 1 a) í b) í c) í 2 x + my = 2 î2 x + my = 2m + 5 î (m - 1) x - my = 2 î ĐS: Bài 6. Giải và biện luận các hệ phương trình sau: ìax + y = b ì y - ax = b ìax + y = a + b a) í b) í c) í î3 x + 2 y = -5 î2 x - 3y = 4 î x + 2y = a Bài 4. ì(a + b) x + (a - b)y = a d) í e) î(2 a - b) x + (2a + b)y = b ĐS: Bài 7. Giải các hệ phương trình sau: ì3 x + y - z = 1 ï a) í2 x - y + 2z = 5 b) ïî x - 2 y - 3z = 0 ĐS: ìax + by = a2 + b2 í îbx + ay = 2ab ìïax - by = a2 - b f) í 2 ïîbx - b y = 4b ì x + 3y + 2 z = 8 ï í2 x + y + z = 6 ïî3 x + y + z = 6 ì x - 3 y + 2 z = -7 ï c) í-2 x + 4 y + 3z = 8 ïî3 x + y - z = 5 Trang 30 Phương trình bậc nhất – bậc hai IX. HỆ PHƯƠNG TRÌNH BẬC HAI HAI ẨN 1. Hệ gồm 1 phương trình bậc nhất và 1 phương trình bậc hai · Từ phương trình bậc nhất rút một ẩn theo ẩn kia. · Thế vào phương trình bậc hai để đưa về phương trình bậc hai một ẩn. · Số nghiệm của hệ tuỳ theo số nghiệm của phương trình bậc hai này. 2. Hệ đối xứng loại 1 ì f ( x , y) = 0 Hệ có dạng: (I) í (với f(x, y) = f(y, x) và g(x, y) = g(y, x)). î g( x , y ) = 0 (Có nghĩa là khi ta hoán vị giữa x và y thì f(x, y) và g(x, y) không thay đổi). · Đặt S = x + y, P = xy. · Đưa hệ phương trình (I) về hệ (II) với các ẩn là S và P. · Giải hệ (II) ta tìm được S và P. · Tìm nghiệm (x, y) bằng cách giải phương trình: X 2 - SX + P = 0 . 3. Hệ đối xứng loại 2 ì f ( x , y) = 0 (1) Hệ có dạng: (I) í (2) î f ( y, x ) = 0 (Có nghĩa là khi hoán vị giữa x và y thì (1) biến thành (2) và ngược lại). · Trừ (1) và (2) vế theo vế ta được: ì f ( x , y) - f ( y, x ) = 0 (3) (I) Û í (1) î f ( x , y) = 0 · Biến đổi (3) về phương trình tích: éx = y (3) Û ( x - y ).g( x , y ) = 0 Û ê . ë g( x , y ) = 0 éì f ( x, y ) = 0 êí x = y · Như vậy, (I) Û ê î . ê ìí f ( x, y ) = 0 êë î g( x , y ) = 0 · Giải các hệ trên ta tìm được nghiệm của hệ (I). 4. Hệ đẳng cấp bậc hai ìïa x 2 + b xy + c y 2 = d 1 1 . Hệ có dạng: (I) í 1 2 1 2 ïîa2 x + b2 xy + c2 y = d2 · Giải hệ khi x = 0 (hoặc y = 0). · Khi x ¹ 0, đặt y = kx . Thế vào hệ (I) ta được hệ theo k và x. Khử x ta tìm được phương trình bậc hai theo k. Giải phương trình này ta tìm được k, từ đó tìm được (x; y). Chú ý: – Ngoài các cách giải thông thường ta còn sử dụng phương pháp hàm số để giải (sẽ học ở lớp 12). – Với các hệ phương trình đối xứng, nếu hệ có nghiệm ( x0 ; y0 ) thì ( y0 ; x0 ) cũng là nghiệm của hệ. Do đó nếu hệ có nghiệm duy nhất thì x0 = y0 . Trang 31 Phương trình bậc nhất – bậc hai Bài 1. Nguy n Công Nh t Giải các hệ phương trình sau: 2 ì 2 ì 2 a) í x + 4 y = 8 b) í x - xy = 24 î x + 2y = 4 î2 x - 3y = 1 2 ì 2 ì3 x - 4 y + 1 = 0 d) í x - 3 xy + y + 2 x + 3y - 6 = 0 e) í î xy = 3( x + y ) - 9 î2 x - y = 3 2 ì g) í y + x = 4 x î2 x + y - 5 = 0 ĐS: ì2 x + 3 y = 5 h) í 2 2 î3 x - y + 2 y = 4 Giải và biện luận các hệ phương trình sau: ìx + y = 6 ìx + y = m a) í 2 b) í 2 2 2 îx + y = m îx - y + 2x = 2 ĐS: 2 ì c) í( x - y ) = 49 î3 x + 4 y = 84 ì2 x + 3y = 2 f) í î xy + x + y + 6 = 0 ì2 x - y = 5 i) í 2 2 î x + xy + y = 7 Bài 2. Giải các hệ phương trình sau: ì x + xy + y = 11 ìx + y = 4 a) í 2 b) í 2 2 2 î x + y - xy - 2( x + y ) = -3 î x + xy + y = 13 ì x y 13 3 3 3 ì 3 ï + = d) í y x 6 e) í x + x y + y = 17 î x + y + xy = 5 ïî x + y = 6 ĐS: ì3 x - 2 y = 1 c) í 2 2 îx + y = m Bài 3. ì xy + x + y = 5 c) í 2 2 îx + y + x + y = 8 ìï x 4 + x 2 y 2 + y 4 = 481 f) í 2 2 ïî x + xy + y = 37 Giải và biện luận các hệ phương trình sau: ì x + y + xy = m ìx + y = m +1 ì( x + 1)( y + 1) = m + 5 a) í 2 b) í 2 c) í 2 2 2 î xy( x + y) = 4m î x + y = 3 - 2m î x y + xy = 2m - m - 3 ĐS: Bài 4. Bài 5. Giải các hệ phương trình sau: ìï x 2 = 3x + 2 y a) í 2 ïî y = 3y + 2 x ì y ïï x - 3y = 4 x d) í x ï y - 3x = 4 y îï ìï x 2 - 2 y 2 = 2 x + y b) í 2 2 ïî y - 2 x = 2 y + x ì y2 + 2 ï3y = ï x2 e) í 2 ï3 x = x + 2 ï y2 î ìï x 3 = 2 x + y c) í 3 ïî y = 2 y + x ì 2 1 ïï2 x = y + y f) í ï2 y 2 = x + 1 ïî x ĐS: Giải và biện luận các hệ phương trình sau: ìï x 2 = 3x + my ìï x (3 - 4 y 2 ) = m(3 - 4m 2 ) ìï xy + x 2 = m( y - 1) a) í 2 b) í c) í 2 2 2 ïî y = 3y + mx ïî y(3 - 4 x ) = m(3 - 4m ) ïî xy + y = m( x - 1) ĐS: Bài 6. Bài 7. Giải các hệ phương trình sau: ìï x 2 - 3 xy + y 2 = -1 a) í 2 2 ïî3 x - xy + 3y = 13 ìï2 x 2 - 4 xy + y 2 = -1 b) í 2 2 ïî3 x + 2 xy + 2 y = 7 Trang 32 ìï y 2 - 3 xy = 4 c) í 2 2 ïî x - 4 xy + y = 1 Phương trình bậc nhất – bậc hai ìï3 x 2 + 5 xy - 4 y 2 = 38 d) í 2 2 ïî5 x - 9 xy - 3 y = 15 ĐS: ìï x 2 - 2 xy + 3y 2 = 9 e) í 2 2 ïî x - 4 xy + 5y = 5 Giải và biện luận các hệ phương trình sau: ìï x 2 + mxy + y 2 = m ìï xy - y 2 = 12 a) í 2 b) í 2 2 ïî x + (m - 1) xy + my = m ïî x - xy = m + 26 ĐS: ìï3 x 2 - 8 xy + 4 y 2 = 0 f) í 2 2 ïî5x - 7 xy - 6 y = 0 Bài 8. Trang 33 ìï x 2 - 4 xy + y 2 = m c) í 2 ïî y - 3 xy = 4 Phương trình bậc nhất – bậc hai Nguy n Công Nh t BÀI TẬP ÔN CHƯƠNG III Giải và biện luận các phương trình sau: Bài 1. a) m 2 x + 4m - 3 = x + m2 b) (a + b)2 x + 2a2 = 2a(a + b) + (a2 + b2 ) x c) a2 x + 2ab = b2 x + a2 + b2 ĐS: d) a(ax + b) = 4ax + b2 - 5 Tìm m để các phương trình sau có nghiệm: Bài 2. m2 x b) - m x = 2m + 1 x -1 2x + m x + m -1 a) =1 x -1 x c) 2mx - 1 x -1 - 2 x -1 = m +1 x -1 d) x - 1 + 2 x - 3 = m ĐS: Bài 3. Giải và biện luận các phương trình sau: a) 2 x 2 + 12 x - 15m = 0 b) x 2 - 2(m - 1) x + m 2 = 0 b) x 2 - mx + m - 1 = 0 ĐS: d) x 2 - 2(m - 2) x + m(m - 3) = 0 Tìm m để phương trình có một nghiệm x0. Tính nghiệm còn lại: 3 a) x 2 - mx + m + 1 = 0; x0 = b) 2 x 2 - 3m 2 x + m = 0; x0 = 1 . 2 ĐS: Bài 5. Trong các phương trình sau, tìm m để: i) PT có hai nghiệm trái dấu. ii) PT có hai nghiệm âm phân biệt. iii) PT có hai nghiệm dương phân biệt. Bài 4. iv) PT có hai nghiệm phân biệt x1, x2 thoả: x13 + x23 = 0 . v) PT có hai nghiệm phân biệt x1, x2 thoả: x12 + x22 = 3 . a) x 2 - 2(m - 2) x + m(m - 3) = 0 b) x 2 + 2(m - 1) x + m 2 = 0 c) x 2 - 2(m + 1) x + m 2 - 2 = 0 d) (m + 2) x 2 - 2(m - 1) x + m - 2 = 0 e) (m + 1) x 2 + 2(m + 4) x + m + 1 = 0 ĐS: a) b) i) ii) iii) iv) v) f) x 2 - 4 x + m + 1 = 0 Bài 6. c) d) e) f) Trong các phương trình sau, hãy: i) Giải và biện luận phương trình. ii) Khi phương trình có hai nghiệm x1, x2 , tìm hệ thức giữa x1, x2 độc lập với m. a) x 2 + (m - 1) x - m = 0 b) x 2 - 2(m - 2) x + m(m - 3) = 0 c) (m + 2) x 2 - 2(m - 1) x + m - 2 = 0 ĐS: d) x 2 - 2(m + 1) x + m 2 - 2 = 0 Trang 34 Phương trình bậc nhất – bậc hai Giải các phương trình sau: Bài 7. a) x 2 + x 2 - 6 = 12 b) x 2 + x 2 + 11 = 31 c) 16 x + 17 = 8 x - 23 d) x 2 - 2 x - 8 = 3( x - 4) f) 51 - 2 x - x 2 = 1 - x h) x + 3 + 1 = 3x - 1 3x 2 - 9 x + 1 + x - 2 = 0 e) g) ( x - 3) x 2 - 4 = x 2 - 9 ĐS: Giải các phương trình sau: Bài 8. a) 4 - 3 10 - 3 x = x - 2 b) x - 5 + x + 3 = 2x + 4 c) 3x + 4 - 2 x - 1 = x + 3 d) x 2 - 3 x + 3 + x 2 - 3x + 6 = 3 e) x + 2 - 2 x - 3 = 3x - 5 f) 3x - 3 - 5 - x = 2 x - 4 g) 2 x + 2 + 2 x + 1 - x + 1 = 4 ĐS: h) x +1 -1 = x - x + 8 x + 2 x -1 - x - 2 x -1 = 2 b) x + 2 x -1 + x - 2 x -1 = x - x2 - 1 + x + x2 - 1 = 2 d) x 2 - x - x 2 - x + 13 = 7 Giải các phương trình sau: Bài 9. a) c) 4 e) x 2 + 2 x 2 - 3 x + 1 = 3 x + 4 f) 2 x 2 + 3 2 x 2 + x + 1 = 9 - x g) x 2 - x 2 - 2 x + 4 = 2 x - 2 ĐS: h) 2 x 2 + 5 x 2 + 3 x + 5 = 23 - 6 x x +3 2 Bài 10. Trong các hệ phương trình sau: i) Tìm số nguyên m để hệ có nghiệm duy nhất là nghiệm nguyên. ii) Khi hệ có nghiệm (x, y), tìm hệ thức giữa x, y độc lập với m. ìmx + 2 y = m + 1 ìmx + y = 3m a) í b) í î x + my = 2m + 1 î2 x + my = 2m - 1 ì x - 2y = 4 - m c) í î2 x + y = 3m + 3 ĐS: ì2 x + y = 5 d) í î2 y - x = 10m + 5 Bài 11. Giải các hệ phương trình sau: ì x + xy + y = -1 a) í 2 2 î x y + y x = -6 ìï x 3 + y3 = 1 d) í 5 5 2 2 ïî x + y = x + y ĐS: ìï x 2 + y 2 = 5 b) í 4 2 2 4 ïî x - x y + y = 13 ìï x 2 + y 2 + xy = 7 e) í 4 4 2 2 ïî x + y + x y = 21 Bài 12. Giải các hệ phương trình sau: Trang 35 ìï x 2 y + y 2 x = 30 c) í 3 3 ïî x + y = 35 ì x + y + xy = 11 f) í 2 2 î x + y + 3( x + y ) = 28 Phương trình bậc nhất – bậc hai Nguy n Công Nh t ì 1 ï( x + y)(1 + xy ) = 5 ï a) í 1 ï( x 2 + y 2 )(1 + ) = 49 2 ïî x y2 ì 1 1 ïx + y + x + y = 4 ï c) í 1 1 ï x 2 + y2 + + =4 ïî x 2 y2 ì y( x 2 + 1) = 2 x ( y 2 + 1) ï æ b) í 2 1 ö 2 x + y 1 + = 24 ç ï ç x 2 y 2 ÷÷ è ø î ( ) ì x y 2 + = ï 2 2 3 ï d) í x + 1 y + 1 ï( x + y )(1 + 1 ) = 6 ïî xy ì 1 ïï xy + xy = 4 f) í ï( x + y ) æç 1 + 1 ö÷ = 5 ïî è xy ø ì2 x 2 y + y 2 x + 2 y + x = 6 xy ï e) í 1 y x xy + + + =4 ï xy x y î ĐS: Bài 13. Giải các hệ phương trình sau: ìï2 x 2 - y 2 = 3x - 2 a) í 2 2 ïî2 y - x = 3y - 2 ìï2 x 2 - y 2 = 3x + 4 d) í 2 2 ïî2 y - x = 3y + 4 ìï x 3 = 2 y + x + 2 b) í 3 ïî y = 2 x + y + 2 ì 3 ï2 x + y = 2 ï x e) í 3 ï2 y + x = ïî y2 ìï x 3 = 3 x + 8y c) í 3 ïî y = 3y + 8x ì 2y ïx = ï 1 - y2 f) í ïy = 2x ïî 1 - x2 ĐS: Bài 14. Giải các hệ phương trình sau: ìï1 + x 3 y3 = 19 x 3 a) í 2 2 ïî y + xy = -6 x ĐS: ìï8 x 3 y 3 + 27 = 18 y 3 b) í 2 2 ïî4 x y + 6 x = y Trang 36 c) Bất đẳng thức – Bất phương trình CHƯƠNG IV BẤT ĐẲNG THỨC VÀ BẤT PHƯƠNG TRÌNH I. BẤT ĐẲNG THỨC 1. Tính chất Điều kiện c>0 c 0, c > 0 n nguyên dương Nội dung a0 a 0, "x Î R \ í- ý î 2a þ a.f(x) > 0, "x Î (–∞; x1) È (x2; +∞) a.f(x) < 0, "x Î (x1; x2) ìa > 0 Nhận xét: · ax 2 + bx + c > 0, "x Î R Û í îD < 0 ìa < 0 · ax 2 + bx + c < 0, "x Î R Û í îD < 0 2. Bất phương trình bậc hai một ẩn ax 2 + bx + c > 0 (hoặc ³ 0; < 0; £ 0) Để giải BPT bậc hai ta áp dụng định lí về dấu của tam thức bậc hai. VẤN ĐỀ 1: Giải bất phương trình, hệ bất phương trình bậc hai một ẩn Bài 1. Xét dấu các biểu thức sau: a) 3 x 2 - 2 x + 1 b) - x 2 + 4 x + 5 c) -4 x 2 + 12 x - 9 d) 3 x 2 - 2 x - 8 e) - x 2 + 2 x - 1 f) 2 x 2 - 7 x + 5 g) (3 x 2 - 10 x + 3)(4 x - 5) h) (3 x 2 - 4 x )(2 x 2 - x - 1) i) ĐS: (3 x 2 - x )(3 - x 2 ) 4x2 + x - 3 Bài 2. Giải các bất phương trình sau: a) 2 x 2 - 5 x + 2 < 0 b) -5 x 2 + 4 x + 12 < 0 c) 16 x 2 + 40 x + 25 > 0 d) -2 x 2 + 3 x - 7 ³ 0 e) 3 x 2 - 4 x + 4 ³ 0 f) x 2 - x - 6 £ 0 g) -3 x 2 - x + 4 ĐS: x 2 + 3x + 5 >0 h) 4 x 2 + 3x - 1 x2 + 5x + 7 >0 i) 5x 2 + 3x - 8 x2 - 7x + 6 0 b) (1 + m) x 2 - 2 mx + 2m £ 0 c) mx 2 - 2 x + 4 > 0 HD: Giải và biện luận BPT bậc hai, ta tiến hành như sau: – Lập bảng xét dấu chung cho a và D. – Dựa vào bảng xét dấu, biện luận nghiệm của BPT. Bài 4. Giải các hệ bất phương trình sau: ìï2 x 2 + 9 x + 7 > 0 a) í 2 ïî x + x - 6 < 0 ìï2 x 2 + x - 6 > 0 b) í 2 ïî3 x - 10 x + 3 ³ 0 Trang 53 ìï-2 x 2 - 5 x + 4 < 0 c) í 2 ïî- x - 3 x + 10 > 0 Bất đẳng thức – Bất phương trình ìx2 + 4x + 3 ³ 0 ï d) í2 x 2 - x - 10 £ 0 ï2 x 2 - 5 x + 3 > 0 î g) -4 £ ĐS: x2 - 2x - 7 x2 + 1 £1 Nguy n Công Nh t ìï- x 2 + 4 x - 7 < 0 e) í 2 ïî x - 2 x - 1 ³ 0 ìï x 2 + x + 5 < 0 f) í 2 ïî x - 6 x + 1 > 0 1 x2 - 2x - 2 h) £ £1 13 x 2 - 5 x + 7 i) -1 < 10 x 2 - 3 x - 2 - x2 + 3x - 2 VẤN ĐỀ 2: Phương trình bậc hai – Tam thức bậc hai Bài 1. Tìm m để các phương trình sau: i) có nghiệm 2 ii) vô nghiệm 2 a) (m - 5) x - 4mx + m - 2 = 0 b) (m - 2) x + 2(2m - 3) x + 5m - 6 = 0 c) (3 - m) x 2 - 2(m + 3) x + m + 2 = 0 d) (1 + m) x 2 - 2mx + 2m = 0 e) (m - 2) x 2 - 4mx + 2m - 6 = 0 ĐS: a) b) có nghiệm vô nghiệm f) (- m 2 + 2m - 3) x 2 + 2(2 - 3m) x - 3 = 0 c) d) e) Bài 2. Tìm m để các bất phương trình sau nghiệm đúng với mọi x: a) 3 x 2 + 2(m - 1) x + m + 4 > 0 b) x 2 + (m + 1) x + 2m + 7 > 0 c) 2 x 2 + (m - 2) x - m + 4 > 0 d) mx 2 + (m - 1) x + m - 1 < 0 e) (m - 1) x 2 - 2(m + 1) x + 3(m - 2) > 0 ĐS: f) 3(m + 6) x 2 - 3(m + 3) x + 2m - 3 > 3 Bài 3. Tìm m để các bất phương trình sau vô nghiệm: a) (m + 2) x 2 - 2(m - 1) x + 4 < 0 b) (m - 3) x 2 + (m + 2) x - 4 > 0 c) (m 2 + 2m - 3) x 2 + 2(m - 1) x + 1 < 0 d) mx 2 + 2(m - 1) x + 4 ³ 0 e) (3 - m) x 2 - 2(2m - 5) x - 2m + 5 > 0 ĐS: f) mx 2 - 4(m + 1) x + m - 5 < 0 Trang 54 f) 0 · Dạng 3: f ( x ) < g( x ) Û ê î Ûí î- g( x ) < f ( x ) < g( x ) ê ìí f ( x ) < 0 êë î- f ( x ) < g( x ) é ì g( x ) < 0 éì f (x) ³ 0 ê í f ( x ) coù nghóa ê í f ( x ) > g( x ) êî î · Dạng 4: f ( x ) > g( x ) Û ê Û ê ì g( x ) ³ 0 ï ê ìí f ( x ) < 0 ê í é f ( x ) < - g( x ) êë î- f ( x ) > g( x ) ê ï ê f ( x ) > g( x ) ëîë Chú ý: · A = A Û A³ 0; A = -A Û A £ 0 é A < -B . A >BÛê ëA > B · A + B = A + B Û AB ³ 0 ; A - B = A + B Û AB £ 0 2. Phương trình – Bất phương trình chứa ẩn trong dấu căn Để giải phương trình, bất phương trình chứa ẩn trong dấu căn ta thường dùng phép nâng luỹ thừa hoặc đặt ẩn phụ để khử dấu căn. ìï g( x ) ³ 0 · Dạng 1: f ( x ) = g( x ) Û í 2 ïî f ( x ) = [ g( x )] ì f ( x ) ³ 0 (hoaëc g( x ) ³ 0) · Dạng 2: f ( x ) = g( x ) Û í î f ( x ) = g( x ) · Với B > 0 ta có: · Dạng 3: · Dạng 4: · Dạng 5: · Dạng 6: A < B Û -B < A < B ; ìït = f ( x ), t ³ 0 a. f ( x ) + b. f ( x ) + c = 0 Û í 2 ïîat + bt + c = 0 ïìu = f ( x ) f ( x ) ± g( x ) = h( x ) . Đặt í ; u, v ³ 0 đưa về hệ u, v. ïîv = g( x ) ì f ( x) ³ 0 f ( x ) < g( x ) Û ïí g( x ) > 0 ï f ( x ) < [ g( x )]2 î é ì g( x ) < 0 êí f ( x) ³ 0 î f ( x ) > g( x ) Û ê ì g( x ) ³ 0 êï ê í f ( x ) > [ g( x )]2 ë ïî Trang 55 Bất đẳng thức – Bất phương trình Nguy n Công Nh t Bài 1. Giải các phương trình sau: a) x 2 - 5 x + 4 = x 2 + 6 x + 5 b) x 2 - 1 = x 2 - 2 x + 8 c) 2 - 3 x 2 - 6 - x 2 = 0 d) 2 x - x - 3 = 3 e) x 2 - 1 = 1 - x f) x2 - 1 + x + 1 =2 x ( x - 2) 1 9 b) x = c) x = ± 2 11 2 Bài 2. Giải các bất phương trình sau: ĐS: a) x = - a) 2 x 2 - 5x - 3 < 0 b) x - 8 > x 2 + 3 x - 4 c) x 2 - 1 - 2 x < 0 d) x 2 + 4 x + 3 > x 2 - 4 x - 5 e) x - 3 - x + 1 < 2 f) x 2 - 3 x + 2 + x 2 > 2 x x2 - 4 x 2x - 5 +1 > 0 x -3 x-2 £1 h) a) 2x - 3 = x - 3 b) 5 x + 10 = 8 - x c) x - 2 x - 5 = 4 d) x2 + 2 x + 4 = 2 - x e) 3x 2 - 9 x + 1 = x - 2 f) g) 3x + 7 - x + 1 = 2 h) x2 + 9 - x2 - 7 = 2 i) x + 5 + 3 x + 6 = 3 2 x + 11 b) g) 2 x + x+2 æ 1ö æ 3ö ĐS: a) ç -3; ÷ È ç 1; ÷ è 2ø è 2ø Bài 3. Giải các phương trình sau: i) 2 x - 5x + 6 ³3 3x 2 - 9 x + 1 = x - 2 21 + x + 21 - x 21 + x - 21 - x ĐS: a) x = 6 b) c) d) x = -1; x = -2 e) f) g) x = -1; x = 3 h) x = ±4 i) Bài 4. Giải các phương trình sau: (nâng luỹ thừa) a) 3 3 3 a) x - 2 + 2x - 5 + x + 2 + 3 2x - 5 = 7 2 b) x + 5 - 4 x +1 + x + 2 - 2 x +1 = 1 c) 2 x - 2 2 x - 1 - 2 2 x + 3 - 4 2 x - 1 + 3 2 x + 8 - 6 2 x - 1 = 4 ĐS: a) x = 15 b) c) Bài 6. Giải các phương trình sau: (đặt ẩn phụ) a) x 2 - 6 x + 9 = 4 x 2 - 6 x + 6 b) ( x + 4)( x + 1) - 3 x 2 + 5 x + 2 = 6 c) ( x - 3)2 + 3 x - 22 = x 2 - 3x + 7 d) ( x + 1)( x + 2) = x 2 + 3 x - 4 ĐS: a) b) x = -7; x = 2 c) x = 6; x = -3 d) Bài 7. Giải các phương trình sau: (đặt hai ẩn phụ) x2 + 9 - x2 - 7 = 2 b) 3 5 x + 7 - 3 5 x - 12 = 1 d) 3 1 + x + 3 1 - x = 2 3 9 - x +1 + 3 7 + x +1 = 4 f) a) c) e) 21 x x + 1 + 3 3 x + 1 = 3 x - 1 c) 3 1 + x + 3 1 - x = 2 x +1 + 3 x + 2 + 3 x + 3 = 0 11 ĐS: a) x = -5; x = -6; x = 2 Bài 5. Giải các phương trình sau: (biến đổi biểu thức dưới căn) d) = 3x 2 + 5 x + 8 - 3 x 2 + 5 x + 1 = 1 3 24 + x - 3 5 + x = 1 Trang 56 Bất đẳng thức – Bất phương trình g) 4 47 - 2 x + 4 35 + 2 x = 4 h) x 2 + 4356 + x - x x 2 + 4356 - x 2 = 5 x 8 c) x = -3; x = 4 d) x = 0 3 Bài 8. Giải các bất phương trình sau: ĐS: a) x = ±4 b) x = 1; x = - a) x 2 + x - 12 < 8 - x b) x 2 - x - 12 < 7 - x c) - x 2 - 4 x + 21 < x + 3 d) x 2 - 3 x - 10 > x - 2 e) 3x 2 + 13 x + 4 ³ x - 2 f) 2x + 6x2 + 1 > x + 1 2 - x > 7 - x - -3 - 2 x h) x + 3 - 7 - x > 2 x - 8 i) é 76 ö ĐS: a) ( -¥; -4] È ê3; ÷ d) ( -¥; -2] È (14; +¥) ë 17 ø Bài 9. Giải các bất phương trình sau: 2x + 3 + x + 2 £ 1 g) a) ( x - 3)(8 - x ) + 26 > - x 2 + 11x b) ( x + 5)( x - 2) + 3 x ( x + 3) > 0 c) ( x + 1)( x + 4) < 5 x 2 + 5 x + 28 d) 3x 2 + 5 x + 7 - 3 x 2 + 5 x + 2 ³ 1 b) -2 x 2 - 15 x + 17 ³0 x +3 d) -x2 + x + 6 -x2 + x + 6 ³ 2x + 5 x+4 ĐS: a) [3;4 ) È ( 7;8] b) (-¥; -4) È (1; +¥) Bài 10. Giải các bất phương trình sau: a) x2 - 4x £2 3- x c) ( x + 3) x 2 - 4 £ x 2 - 9 ĐS: Bài 11. Giải các bất phương trình sau: 3 a) x + 2 £ x 2 + 8 ĐS: b) 3 3 2 x 2 + 1 ³ 3x 2 - 1 Trang 57 c) 3 x +1 > x - 3 Bất đẳng thức – Bất phương trình Nguy n Công Nh t BÀI TẬP ÔN CHƯƠNG IV Bài 1. Chứng minh các bất đẳng thức sau: a) a3 + b3 + c3 ³ a + b + c , với a, b, c > 0 và abc = 1 . a+b+c a+b+c a+b+c b) + + ³ 9 , với a, b, c > 0. a b c æ 1 1 1ö 1 1 1 c) + + ³ 2 ç + + ÷ , với a, b, c là 3 cạnh của 1 tam giác, p nửa chu vi. p-a p-b p-c èa b cø d) a b - 1 + b a - 1 £ ab , với a ³ 1, b ³ 1. 3 HD: a) Áp dụng BĐT Cô–si: a3 + b3 + c3 ³ 3 a3b3c3 = 3 Þ 2(a3 + b3 + c3 ) ³ 6 (1) 3 a3 + 1 + 1 ³ 3 a3 Þ a3 + 2 ³ 3a (2). Tương tự: b3 + 2 ³ 3b (3), c3 + 2 ³ 3c (4). Cộng các BĐT (1), (2), (3), (4) vế theo vế ta được đpcm. æb aö æb cö æc aö b) BĐT Û ç + ÷ + ç + ÷ + ç + ÷ ³ 6 . Dễ dàng chứng minh. èa bø èc bø èa cø 1 1 4 1 1 4 4 c) Áp dụng BĐT: + ³ , ta được: + ³ = . x y x+y p-a p-b p-a+ p-b c Tương tự: 1 1 4 1 1 4 + ³ ; + ³ . Cộng các BĐT Þ đpcm. p-b p-c a p-c p-a b d) Áp dụng BĐT Cô–si: a b - 1 = a . ab - a £ a + ab - a ab = . 2 2 ab . Cộng 2 BĐT ta được đpcm. Dấu "=" xảy ra Û a = b = 2. 2 Bài 2. Tìm GTNN của các biểu thức sau: 1 a) A = x + , với x > 1. x -1 4 1 5 b) B = + , với x, y > 0 và x + y = . x 4y 4 Tương tự: b a - 1 £ c) C = a + b + 1 1 + , với a, b > 0 và a + b £ 1 . a b d) D = a3 + b3 + c3 , với a, b, c > 0 và ab + bc + ca = 3 . 1 HD: a) Áp dụng BĐT Cô–si: A = ( x - 1) + +1 ³ 2 +1 = 3. x -1 Dấu "=" xảy ra Û x = 2. Vậy minA = 3. b) B = 4 1 4 1 + 4x + + 4 y - 5 ³ 2 .4 x + 2 .4 y - 5 = 5 . x 4y x 4y 1 . Vậy minB = 5. 4 1 1 4 3 4 1 3 c) Ta có + ³ Þ C ³a+b+ =a+b+ + ³ 2+ ³ 5. a b a+b a+b a+b a+b a+b 1 Dấu "=" xảy ra Û a = b = . Vậy minC = 5. 2 Dấu "=" xảy ra Û x = 1; y = d) Áp dụng BĐT Cô–si: a3 + b3 + 1 ³ 3ab , b3 + c3 + 1 ³ 3bc , c3 + a3 + 1 ³ 3ca . Þ 2(a3 + b3 + c3 ) + 3 ³ 3(ab + bc + ca) = 9 Þ a3 + b3 + c3 ³ 3 . Trang 58 Bất đẳng thức – Bất phương trình Dấu "=" xảy ra Û a = b = c = 1. Vậy minD = 3. Bài 3. Tìm GTLN của các biểu thức sau: a) A = a + 1 + b + 1 , với a, b ³ –1 và a + b = 1 . 1 b) B = x 2 (1 - 2 x ) , với 0 < x < . 2 1 c) C = ( x + 1)(1 - 2 x ) , với -1 < x < . 2 HD: a) Áp dụng BĐT (B) cho 4 số 1,1, a + 1, b + 1 ta được: A = 1. a + 1 + 1. b + 1 £ (1 + 1)(a + 1 + b + 1) = 6 . Dấu "=" xảy ra Û a = b = Þ maxA = 6. 3 æ x + x +1- 2x ö 1 b) Áp dụng BĐT Cô–si: B = x.x (1 - 2 x ) £ ç . ÷ = è 3 ø 27 1 1 . Vậy maxB = . 3 27 2 1 1 æ 2x + 2 + 1 - 2x ö 9 c) Áp dụng BĐT Cô–si: C = (2 x + 2)(1 - 2 x ) £ ç ÷ = . 2 2è 2 ø 8 1 9 Dấu "=" xảy ra Û x = - . Vậy maxC = . 4 8 Bài 4. Tìm m để các hệ bất phương trình sau có nghiệm: 2 ì ì 2 a) í x + 4m £ 2mx + 1 b) í x - 3 x - 4 £ 0 î3 x + 2 > 2 x - 1 î(m - 1) x - 2 ³ 0 ì ì7 x - 2 ³ -4 x + 19 c) í d) í 2 x + 1 > x - 2 î2 x - 3m + 2 < 0 îm + x > 2 ĐS: Bài 5. Tìm m để các hệ bất phương trình sau vô nghiệm: 2 ì ì 2 a) ímx + 9 < 3 x + m b) í x + 10 x + 16 £ 0 î4 x + 1 < - x + 6 îmx > 3m + 1 ĐS: Bài 6. Giải các bất phương trình sau: a) 2x - 5 < 1 x -3 x2 - 6 x - 7 2 1 2x -1 c) ³ x2 - x + 1 x + 1 x3 + 1 ĐS: b) x 2 - 5x + 6 ³ x +1 x x2 + 5x + 6 2 1 1 d) + £0 x x -1 x +1 Bài 7. Tìm m để các phương trình sau có nghiệm: a) (m - 1) x 2 - 2(m + 3) x - m + 2 = 0 b) (m - 1) x 2 + 2(m - 3) x + m + 3 = 0 ĐS: Bài 8. Tìm m để các biểu thức sau luôn không âm: a) (3m + 1) x 2 - (3m + 1) x + m + 4 b) (m + 1) x 2 - 2(m - 1) x + 3m - 3 ĐS: Bài 9. Tìm m để các biểu thức sau luôn âm: Trang 59 1 . 2 Bất đẳng thức – Bất phương trình Nguy n Công Nh t a) (m - 4) x 2 + (m + 1) x + 2 m - 1 b) (m 2 + 4m - 5) x 2 - 2(m - 1) x + 2 ĐS: Bài 10. Tìm m để các bất phương trình sau nghiệm đúng với mọi x: a) c) x 2 - 8 x + 20 mx 2 + 2(m + 1) x + 9m + 4 x 2 + mx - 1 2x2 - 2x + 3 ĐS: x 2 - 5 x + 9 h) 2 x + 1 < x - 2 + 3 x + 1 Bài 15. Giải các phương trình sau: a) x - 2 x + 3 = 0 b) 2 x + 3 + x + 1 = 3 x + 2 (2 x + 3)( x + 1) - 16 c) x + 4 - 1- x = 1- 2x d) x + 1 + 4 - x + ( x + 1)(4 - x ) = 5 e) 4x - 1 + 4x2 -1 = 1 f) 3x - 2 + x - 1 = 4 x - 9 + 2 3 x 2 - 5 x + 2 g) ( x + 5)(2 - x ) = 3 x 2 + 3 x h) x ( x - 4) - x 2 + 4 x + ( x - 2)2 = 2 i) x 2 + x 2 + 11 = 31 ĐS: k) x + 9 - x = - x2 + 9x + 9 Trang 60 Bất đẳng thức – Bất phương trình Bài 16. Giải các bất phương trình sau - x 2 - 8 x - 12 > x + 4 a) d) 3(4 x 2 - 9) 2 3x - 3 £ 2x + 3 b) 5 x 2 + 61x < 4 x + 2 e) ( x - 3) x 2 + 4 £ x 2 - 9 ĐS: Trang 61 c) f) 2 - x + 4x - 3 ³2 x 9x2 - 4 2 5x - 1 £ 3x + 2 Thống kê Nguy n Công Nh t CHƯƠNG V THỐNG KÊ I. Một số khái niệm · Một tập con hữu hạn các đơn vị điều tra đgl một mẫu. · Số phần tử của một mẫu đgl kích thước mẫu. · Các giá trị của dấu hiệu thu được trên mẫu đgl một mẫu số liệu. II. Trình bày một mẫu số liệu · Tần số của một giá trị là số lần xuất hiện của giá trị đó trong mẫu số liệu. · Tần suất fi của giá trị xi là tỉ số giữa tần số ni và kích thước mẫu N: fi = ni (thường viết tần suất dưới dạng %) N · Bảng phân bố tần số – tần suất · Bảng phân bố tần số – tần suất ghép lớp Giá trị Tần số Tần suất (%) Lớp Tần số Tần suất (%) x1 n1 f1 [x1; x2) n1 f1 x2 n2 f2 [x2; x3) n2 f2 … … … … … … xk nk fk [xk; xk+1) nk fk N 100 (%) N 100 (%) III. Biểu đồ · Biểu đồ hình cột · Biểu đồ hình quạt · Đường gấp khúc IV. Các số đặc trưng của mẫu số liệu 1. Số trung bình · Với mẫu số liệu kích thước N là { x1, x2 ,..., x N } : x= x1 + x2 + ... + x N N · Với mẫu số liệu được cho bởi bảng phân bố tần số: n x + n x + ... + nk xk x= 1 1 2 2 N · Với mẫu số liệu được cho bởi bảng phân bố tần số ghép lớp: n c + n c + ... + nk ck x= 11 2 2 (ci là giá trị đại diện của lớp thứ i) N 2. Số trung vị Giả sử ta có một mẫu gồm N số liệu được sắp xếp theo thứ tự không giảm (hoặc không tăng). Khi đó số trung vị Me là: – Số đứng giữa nếu N lẻ; – Trung bình cộng của hai số đứng giữa nếu N chẵn. 3. Mốt Mốt của một bảng phân bố tần số là giá trị có tần số lớn nhất và được kí hiệu là MO . Chú ý: – Số trung bình của mẫu số liệu được dùng làm đại diện cho các số liệu của mẫu. – Nếu các số liệu trong mẫu có sự chênh lệch quá lớn thì dùng số trung vị làm đại diện cho các số liệu của mẫu. – Nếu quan tâm đến giá trị có tần số lớn nhất thì dùng mốt làm đại diện. Một mẫu số liệu có thể có nhiều mốt. 4. Phương sai và độ lệch chuẩn Để đo mức độ chênh lệch (độ phân tán) giữa các giá trị của mẫu số liệu so với số Trang 62 Thống kê trung bình ta dùng phương sai s2 và độ lệch chuẩn s = s2 . · Với mẫu số liệu kích thước N là { x1, x2 ,..., x N } : 1 s = N 2 N 1 å ( xi - x ) = N i =1 2 N å i =1 xi2 1 æN ö ç å xi ÷ N 2 è i =1 ø 2 2 = x - ( x )2 · Với mẫu số liệu được cho bởi bảng phân bố tần số, tần suất: ö 1 æ k ni xi ÷ ç å ni ( xi - x ) å å N 2 è i =1 i =1 i =1 ø 2 k k æ k ö = å fi ( xi -x )2 = å fi xi2 - ç å fi xi ÷ i =1 i =1 è i =1 ø · Với mẫu số liệu được cho bởi bảng phân bố tần số, tần suất ghép lớp: 1 s = N 2 k 2 1 = N k 2 ni xi2 2 ö 1 æ k å ni (ci - x ) å - 2 ç å ni ci ÷ N è i =1 i =1 i =1 ø 2 k k æ k ö = å fi (ci -x )2 = å fi ci2 - ç å fi ci ÷ i =1 i =1 è i =1 ø (ci, ni, fi là giá trị đại diện, tần số, tần suất của lớp thứ i; N là số các số liệu thống kê N = n1 + n2 + ... + nk ) Chú ý: Phương sai và độ lệch chuẩn càng lớn thì độ phân tán (so với số trung bình) của các số liệu thống kê càng lớn. 1 s = N 2 k 2 1 = N k ni ci2 Bài 1. Trong các mẫu số liệu dưới đây: i) Cho biết dấu hiệu và đơn vị điều tra là gì? Kích thước mẫu là bao nhiêu? ii) Lập bảng phân bố tần số, tần suất. Nhận xét. iii) Vẽ biểu đồ tần số, tần suất. iv) Tính số trung bình, số trung vị, mốt. v) Tính phương sai và độ lệch chuẩn. Nhận xét. 1) Tuổi thọ của 30 bóng đèn được thắp thử (đơn vị: giờ) 1180 1150 1190 1170 1180 1170 1160 1170 1160 1150 1190 1180 1170 1170 1170 1190 1170 1170 1170 1180 1170 1160 1160 1160 1170 1160 1180 1180 1150 1170 2) Năng suất lúa hè thu (tạ/ha) năm 1998 của 31 tỉnh 30 30 25 25 35 45 40 40 35 45 25 45 30 30 30 40 30 25 45 45 35 35 30 40 40 40 35 35 35 35 35 3) Số con của 40 gia đình ở huyện A. 2 4 3 2 0 2 2 3 4 5 2 2 5 2 1 2 2 2 3 2 5 2 7 3 4 2 2 2 3 2 3 5 2 1 2 4 4 3 4 3 4) Điện năng tiêu thụ trong một tháng (kW/h) của 30 gia đình ở một khu phố A. 165 85 65 65 70 50 45 100 45 100 100 100 100 90 53 70 141 42 50 150 40 70 84 59 75 57 133 45 65 75 Trang 63 Thống kê Nguy n Công Nh t 5) Số học sinh giỏi của 30 lớp ở một trường THPT. 0 2 1 0 0 3 0 0 1 1 0 1 6 6 0 1 5 2 4 5 1 0 1 2 4 0 3 3 1 0 6) Nhiệt độ của 24 tỉnh, thành phố ở Việt Nam vào một ngày của tháng 7 (đơn vị: độ) 36 30 31 32 31 40 37 29 41 37 35 34 34 35 32 33 35 33 33 31 34 34 32 35 7) Tốc độ (km/h) của 30 chiếc xe môtô ghi ở một trạm kiểm soát giao thông. 40 58 60 75 45 70 60 49 60 75 52 41 70 65 60 42 80 65 58 55 65 75 40 55 68 70 52 55 60 70 8) Kết quả điểm thi môn Văn của hai lớp 10A, 10B ở một trường THPT. Lớp 10A Điểm thi 5 6 7 8 9 10 Cộng Tần số 1 9 12 14 1 3 40 Lớp 10B Điểm thi 6 7 8 9 Cộng Tần số 8 18 10 4 40 9) Tiền lương hàng tháng của 30 công nhân ở một xưởng may. Tiền lương 300 500 700 800 900 1000 Cộng Tần số 3 5 6 5 6 5 30 10) Một nhà nghiên cứu ghi lại tuổi của 30 bệnh nhân mắc bệnh đau mắt hột. 21 17 22 18 20 17 15 13 15 20 15 12 18 17 17 21 15 12 18 16 23 14 18 19 13 16 19 18 ĐS: Me MO x s2 1) 2) 3) 4) 5) 6) 7) 8) 9) 10) 1170 1170 25 17 s 1170 Bài 2. Trong các mẫu số liệu dưới đây: i) Cho biết dấu hiệu và đơn vị điều tra là gì? Kích thước mẫu là bao nhiêu? ii) Lập bảng phân bố tần số, tần suất ghép lớp. Nhận xét. iii) Vẽ biểu đồ tần số, tần suất. iv) Tính số trung bình, số trung vị, mốt. v) Tính phương sai và độ lệch chuẩn. Nhận xét. 1) Khối lượng của 30 củ khoai tây thu hoạch được ở nông trường T (đơn vị: g). 90 73 88 99 100 102 101 96 79 93 81 94 96 93 95 82 90 106 103 116 109 108 112 87 74 91 84 97 85 92 Với các lớp: [70; 80), [80; 90), [90; 100), [100; 110), [110; 120]. 2) Chiều cao của 35 cây bạch đàn (đơn vị: m). 6,6 7,5 8,2 8,2 7,8 7,9 9,0 8,9 8,2 7,2 7,5 8,3 7,4 8,7 7,7 7,0 9,4 8,7 8,0 7,7 7,8 8,3 8,6 8,1 8,1 9,5 6,9 8,0 7,6 7,9 7,3 8,5 8,4 8,0 8,8 Với các lớp: [6,5; 7,0), [7,0; 7,5), [7,5; 8,0), [8,0; 8,5), [8,5; 9,0), [9,0; 9,5]. Trang 64 Thống kê 3) Số phiếu dự đoán đúng của 25 trận bóng đá học sinh. 54 75 121 142 154 159 171 189 203 211 225 247 251 259 264 278 290 305 315 322 355 367 388 450 490 Với các lớp: [50; 124], [125; 199], … (độ dài mỗi đoạn là 74). 4) Doanh thu của 50 cửa hàng của một công ti trong một tháng (đơn vị: triệu đồng). 102 121 129 114 95 88 109 147 118 148 128 71 93 67 62 57 103 135 97 166 83 114 66 156 88 64 49 101 79 120 75 113 155 48 104 112 79 87 88 141 55 123 152 60 83 144 84 95 90 27 Với các lớp: [26,5; 48,5), [48,5; 70,5), … (độ dài mỗi khoảng là 22). 5) Điểm thi môn Toán của 60 học sinh lớp 10. 1 5 4 8 2 9 4 5 3 2 7 2 7 10 0 2 6 3 7 5 9 10 10 7 9 0 5 3 8 2 4 1 3 6 0 10 3 3 0 8 6 4 1 6 8 2 5 2 1 5 1 8 5 7 2 4 6 3 4 2 Với các lớp: [0;2), [2; 4), …, [8;10]. 6) Số điện tiêu thụ của 30 hộ ở một khu dân cư trong một tháng như sau (đơn vị: kW): 50 47 30 65 63 70 38 34 48 53 33 39 32 40 50 55 50 61 37 37 43 35 65 60 31 33 41 45 55 59 Với các lớp: [30;35), [35; 40), …, [65;70]. 7) Số cuộn phim mà 40 nhà nhiếp ảnh nghiệp dư sử dụng trong một tháng. 5 3 3 1 4 3 4 3 6 8 4 2 4 6 8 9 6 2 10 11 15 1 2 5 13 7 7 2 4 9 3 8 8 10 14 16 17 6 6 12 Với các lớp: [0; 2], [3; 5], …, [15; 17]. 8) Số người đến thư viện đọc sách buổi tối trong 30 ngày của tháng 9 ở một thư viện. 85 81 65 58 47 30 51 92 85 42 55 37 31 82 63 33 44 93 77 57 44 74 63 67 46 73 52 53 47 35 Với các lớp: [25; 34], [35; 44], …, [85; 94] (độ dài mỗi đoạn bằng 9). 9) Số tiền điện phải trả của 50 gia đình trong một tháng ở một khu phố (đơn vị: nghìn đồng) Lớp [375; 449] [450; 524] [525; 599] [600; 674] [675; 749] [750; 825] Tần số 6 15 10 6 9 4 10) Khối lượng của 30 củ khoai tây thu hoạch ở một nông trường (đơn vị: gam). Lớp [70; 80) [80; 90) [90; 100) [100; 110) [110; 120) Tần số 3 6 12 6 3 ĐS: Me MO x s s2 1) 2) 3) 4) 5) 6) 7) 8) 9) 10) Trang 65 Lượng giác Nguy n Công Nh t cos a = x = OH sin a = y = OK sin a tan a = = AT cos a cos a cot a = = BS sin a sin I. Giá trị lượng giác của góc (cung) lượng giác 1. Định nghĩa các giá trị lượng giác Cho (OA, OM ) = a . Giả sử M ( x; y) . tang CHƯƠNG VI GÓC – CUNG LƯỢNG GIÁC CÔNG THỨC LƯỢNG GIÁC B K æ ö p ç a ¹ + kp ÷ è 2 ø T cotang S M a H O (a ¹ kp ) A cosin Nhận xét: · "a , - 1 £ cos a £ 1; - 1 £ sin a £ 1 · tana xác định khi a ¹ p + kp , k Î Z 2 · cota xác định khi a ¹ kp , k Î Z · sin(a + k 2p ) = sin a · tan(a + kp ) = tan a cos(a + k 2p ) = cos a cot(a + kp ) = cot a 2. Dấu của các giá trị lượng giác Phần tư Giá trị lượng giác cosa sina tana cota I II III IV + + + + – + – – – – + + + – – – 3. Giá trị lượng giác của các góc đặc biệt 0 00 p 6 300 p 4 p 3 p 2 2p 3 3p 4 p 3p 2 2p 450 600 900 1200 1350 1800 2700 3600 3 2 2 2 0 –1 0 –1 0 1 sin 0 1 2 2 2 3 2 1 cos 1 3 2 2 2 1 2 0 - tan 0 3 3 1 3 P - 3 –1 0 P 0 cot P 3 1 3 3 0 - 3 3 –1 P 0 P Trang 66 1 2 - 2 2 Lượng giác 4. Hệ thức cơ bản: sin 2a + cos2a = 1 ; tana .cota = 1 ; 1 + tan2 a = 1 cos2 a 5. Giá trị lượng giác của các góc có liên quan đặc biệt Góc đối nhau Góc bù nhau cos(-a ) = cos a sin(p - a ) = sin a sin(-a ) = - sin a cos(p - a ) = - cos a tan(-a ) = - tan a tan(p - a ) = - tan a cot(-a ) = - cot a cot(p - a ) = - cot a Góc hơn kém p ; 1 + cot 2 a = 1 sin2 a Góc phụ nhau æp ö sin ç - a ÷ = cos a è2 ø æp ö cos ç - a ÷ = sin a è2 ø æp ö tan ç - a ÷ = cot a è2 ø æp ö cot ç - a ÷ = tan a è2 ø Góc hơn kém p 2 sin(p + a ) = - sin a æp ö sin ç + a ÷ = cos a è2 ø cos(p + a ) = - cos a æp ö cos ç + a ÷ = - sin a è2 ø tan(p + a ) = tan a æp ö tan ç + a ÷ = - cot a è2 ø cot(p + a ) = cot a æp ö cot ç + a ÷ = - tan a è2 ø II. Công thức lượng giác 1. Công thức cộng sin(a + b) = sin a.cos b + sin b.cos a sin(a - b) = sin a.cos b - sin b.cos a cos(a + b) = cos a.cos b - sin a.sin b cos(a - b) = cos a.cos b + sin a.sin b Hệ quả: tan a + tan b 1 - tan a.tan b tan a - tan b tan(a - b) = 1 + tan a.tan b tan(a + b) = æp ö 1 + tan a tan ç + a ÷ = , è4 ø 1 - tan a æp ö 1 - tan a tan ç - a ÷ = è4 ø 1 + tan a 2. Công thức nhân đôi sin 2a = 2 sin a .cos a cos 2a = cos2 a - sin2 a = 2 cos2 a - 1 = 1 - 2sin 2 a tan 2a = 2 tan a 1 - tan 2 a ; Trang 67 cot 2 a - 1 cot 2a = 2 cot a Lượng giác Nguy n Công Nh t Công thức hạ bậc Công thức nhân ba (*) 1 - cos 2a 2 1 + cos 2a 2 cos a = 2 1 - cos 2a 2 tan a = 1 + cos 2a sin 3a = 3sin a - 4sin3 a cos3a = 4 cos3 a - 3cos a 3tan a - tan3 a tan 3a = 1 - 3tan 2 a sin2 a = 3. Công thức biến đổi tổng thành tích sin(a + b) cos a.cos b sin(a - b) tan a - tan b = cos a.cos b a+b a-b .cos 2 2 a+b a-b cos a - cos b = - 2sin .sin 2 2 a+b a-b sin a + sin b = 2sin .cos 2 2 a+b a-b sin a - sin b = 2 cos .sin 2 2 cos a + cos b = 2 cos tan a + tan b = cot a + cot b = sin(a + b) sin a.sin b cot a - cot b = sin(b - a) sin a.sin b æ æ pö pö sin a + cos a = 2.sin ç a + ÷ = 2.cos ç a - ÷ 4ø 4ø è è æ æ pö pö sin a - cosa = 2 sin ç a - ÷ = - 2 cos ç a + ÷ è 4ø è 4ø 4. Công thức biến đổi tích thành tổng 1 é cos(a - b) + cos(a + b)ùû 2ë 1 sin a.sin b = éë cos(a - b) - cos(a + b)ùû 2 1 sin a.cos b = éësin(a - b) + sin(a + b) ùû 2 cos a.cos b = Trang 68 Lượng giác VẤN ĐỀ 1: Dấu của các giá trị lượng giác Để xác định dấu của các giá trị lượng giác của một cung (góc) ta xác định điểm ngọn của cung (tia cuối của góc) thuộc góc phần tư nào và áp dụng bảng xét dấu các GTLG. Bài 1. Xác định dấu của các biểu thức sau: 22p 7 4p p 4p 9p d) D = cos .sin .tan .cot 5 3 3 5 a) A = sin 500.cos(-3000 ) b) B = sin 2150.tan æ 2p ö 3p .sin ç ÷ 5 è 3 ø ĐS: A > 0; B > 0; c) C = cot Bài 2. Cho 0 0 < a < 900 . Xét dấu của các biểu thức sau: a) A = sin(a + 900 ) b) B = cos(a - 450 ) c) C = cos(2700 - a ) d) D = cos(2a + 90 0 ) ĐS: p Bài 3. Cho 0 < a < . Xét dấu của các biểu thức sau: 2 a) A = cos(a + p ) b) B = tan(a - p ) æ æ 2p ö 3p ö c) C = sin ç a + d) D = cos ç a ÷ ÷ è 5 ø è 8 ø ĐS: Bài 4. Cho tam giác ABC. Xét dấu của các biểu thức sau: a) M = sin A + sin B + sin C b) N = sin A.sin B.sin C A B C A B C c) P = cos .cos .cos d) Q = tan + tan + tan 2 2 2 2 2 2 ĐS: VẤN ĐỀ 2: Tính các giá trị lượng giác của một góc (cung) Ta sử dụng các hệ thức liên quan giữa các giá trị lượng giác của một góc, để từ giá trị lượng giác đã biết suy ra các giá trị lượng giác chưa biết. I. Cho biết một GTLG, tính các GTLG còn lại 1. Cho biết sina, tính cosa, tana, cota · Từ sin2 a + cos2 a = 1 Þ cos a = ± 1 - sin2 a . – Nếu a thuộc góc phần tư I hoặc IV thì cos a = 1 - sin2 a . – Nếu a thuộc góc phần tư II hoặc III thì cos a = - 1 - sin2 a . sin a 1 · Tính tan a = ; cot a = . cos a tan a 2. Cho biết cosa, tính sina, tana, cota · Từ sin2 a + cos2 a = 1 Þ sin a = ± 1 - cos2 a . – Nếu a thuộc góc phần tư I hoặc II thì sin a = 1 - cos2 a . – Nếu a thuộc góc phần tư III hoặc IV thì sin a = - 1 - cos2 a . sin a 1 · Tính tan a = ; cot a = . cos a tan a 3. Cho biết tana, tính sina, cosa, cota Trang 69 Lượng giác Nguy n Công Nh t · Tính cot a = · Từ 1 2 cos a 1 . tan a = 1 + tan2 a Þ cos a = ± 1 1 + tan2 a . – Nếu a thuộc góc phần tư I hoặc IV thì cos a = 1 2 . 1 + tan a 1 – Nếu a thuộc góc phần tư II hoặc III thì cos a = . 2 1 + tan a · Tính sin a = tan a .cos a . 4. Cho biết cota, tính sina, cosa, tana 1 · Tính tan a = . cot a 1 1 · Từ = 1 + cot 2 a Þ sin a = ± . 2 sin a 1 + cot 2 a 1 – Nếu a thuộc góc phần tư I hoặc II thì sin a = . 2 1 + cot a 1 – Nếu a thuộc góc phần tư III hoặc IV thì sin a = . 2 1 + cot a II. Cho biết một giá trị lượng giác, tính giá trị của một biểu thức · Cách 1: Từ GTLG đã biết, tính các GTLG có trong biểu thức, rồi thay vào biểu thức. · Cách 2: Biến đổi biểu thức cần tính theo GTLG đã biết III. Tính giá trị một biểu thức lượng giác khi biết tổng – hiệu các GTLG Ta thường sử dụng các hằng đẳng thức để biến đổi: A2 + B 2 = ( A + B)2 - 2 AB A 4 + B 4 = ( A2 + B 2 )2 - 2 A2 B 2 A3 + B3 = ( A + B)( A2 - AB + B 2 ) A3 - B3 = ( A - B)( A2 + AB + B 2 ) IV. Tính giá trị của biểu thức bằng cách giải phương trình · Đặt t = sin2 x, 0 £ t £ 1 Þ cos2 x = 1 - t . Thế vào giả thiết, tìm được t. Biểu diễn biểu thức cần tính theo t và thay giá trị của t vào để tính. · Thiết lập phương trình bậc hai: t 2 - St + P = 0 với S = u + v; P = uv . Từ đó tìm u,v. Bài 1. Cho biết một GTLG, tính các GTLG còn lại, với: a) cos a = 4 , 270 0 < a < 360 0 5 5 p , < a 0 . Suy ra: x +1 æ x +2 x+2ö æ x+6 x+6ö - x -4 = ç ÷+ç ÷2 ø è x +7 +3 2 ø x+2 +2 x +7 +3 è x+2 +2 1 b (c, b Î N) nên c ≥ b + 1 thành thử: S= a c 1 b +1 b2 + b + 50 S= + ≥ + = b d b 50 50 b Vậy BĐT của đề ra đã được chứng minh. ìa = 1 ï Dấu “=” xảy ra Û íd = 50 ïîc = b + 1 b2 + b + 50 b 1 1 = + + và xét hàm số có biến số liên tục x: 50 b 50 b 50 x 1 1 f(x) = + + (2 ≤ x ≤ 48) 50 x 50 ì 2 1 1 x 2 - 50 f¢(x) = = ; f¢(x) = 0 í x = 50 Û x = 5 2 50 x 2 50 x 2 î2 £ x £ 48 Bảng biến thiên: Để tìm minS, ta đặt 5 2 b2 + b + 50 (2 ≤ b ≤ 48, b Î N) 50 b Từ BBT suy ra khi b biến thiên từ 2 đến 7, f(b) giảm rồi chuyển sang tăng khi b biến thiên từ 8 đến 48. Suy ra minf(b) = min[f(7); f(8)]. 49 + 57 53 64 + 58 61 53 Ta có f(7) = = ; f(8) = = > 350 175 400 200 175 ìa = 1 ïb = 7 53 Vậy minS = khi í 175 ïc = 8 ïîd = 50 5 Baøi 2. (ĐH 2002B–db1) Giả sử x, y là hai số dương thay đổi thoả mãn điều kiện x + y = . 4 4 1 Tìm giá trị nhỏ nhất của biểu thức: S= + . x 4y Chuyển về biểu thức f(b) = ĐS: Ta có: S = 4 1 4 1 + 4x + + 4 y - 5 ³ 2 .4 x + 2 .4 y - 5 = 5 . x 4y x 4y Dấu "=" xảy ra Û x = 1; y = 1 . Vậy minB = 5. 4 Trang 94 Đề thi Tốt nghiệp – Đại học 3 . Gọi a, b, c lần lượt là độ 2 dài các cạnh BC, CA, AB và ha, hb, hc tương ứng là độ dài các đường cao kẻ từ các đỉnh A, B, C. Chứng minh rằng: æ 1 1 1 öæ 1 1 1 ö ç + + ÷ çç + + ÷÷ ³ 3 è a b c ø è ha hb hc ø Baøi 3. (ĐH 2002D–db1) Cho tam giác ABC có diện tích bằng 1 1 1 aha = bhb = chc . 2 2 2 2S 2S 2S 1 1 1 1 Þ ha = ; hb = ; hc = Þ + + = (a + b + c) a b c ha hb hc 2S ĐS: Áp dụng công thức diện tích tam giác: S = æ 1 1 1 öæ 1 1 1 ö 1 æ1 1 1ö Þ ç + + ÷ ç + + ÷ = (a + b + c) ç + + ÷ è a b c ø è ha hb hc ø 2S èa b cø æ1 1 1ö 3 Áp dụng BĐT Cô–si ta có: (a + b + c) ç + + ÷ ³ 9 và chú ý S = , do đó: 2 èa b cø æ 1 1 1 öæ 1 1 1 ö 9 ç + + ÷ çç + + ÷÷ ³ = 3 è a b c ø è ha hb hc ø 3 Baøi 4. (ĐH 2003A) Cho x, y, z là 3 số dương và x + y + z £ 1 . Chứng minh rằng: x2 + 1 2 + y2 + 1 x y ĐS: Áp dụng BĐT (B), ta có: 2 + z2 + 2 1 z2 æ 2 1 ö 2 æ 9ö 2 ç x + 2 ÷ (1 + 9 ) ³ ç x + ÷ Þ xø è x ø è 1 1 æ 9ö Tương tự ta có: y 2 + 2 ³ çy + ÷ yø 82 è y ³ 82 x2 + 1 x 2 ³ (2), 1 æ 9ö çx+ ÷ xø 82 è z2 + 1 z2 ³ (1) 1 æ 9ö çz+ ÷ zø 82 è (3) Từ (1), (2), (3) suy ra: P³ ³ æ 1 1 1 öù 1 é 1 é 1 æ 1 1 1 ö 80 æ 1 1 1 ö ù ê ( x + y + z) + 9 ç + + ÷ ú = ê ( x + y + z) + ç + + ÷ + ç + + ÷ ú 9 è x y z ø 9 è x y z øû 82 ë 82 ë è x y z øû ù æ 1 1 1 ö 80 1 é2 9 ê ( x + y + z) ç + + ÷ + . ú ³ 82 . 82 ëê 3 è x y z ø 9 x + y + z ûú 1 3 Dấu "=" xảy ra Û x = y = z = . Baøi 5. (ĐH 2005A) Cho x, y, z là các số dương thoả mãn : 1 1 1 + + = 4 . Chứng minh rằng: x y z 1 1 1 + + £1 2 x+y+z x + 2 y + z x + y + 2z ĐS: Áp dụng bất đẳng thức 1 1 4 + ³ ( x > 0, y > 0) , ta có: x y x+y 1 1 æ 1 1 1 1ö 1 æ 2 1 1ö £ ç + + + ÷ = ç + + ÷ (1) 2 x + y + z 16 è x x y z ø 16 è x y z ø 1 1 æ 1 2 1ö 1 1 æ1 1 2ö Tương tự: £ ç + + ÷ (2), £ ç + + ÷ x + 2 y + z 16 è x y z ø x + y + 2 z 16 è x y z ø Trang 95 (3) Đề thi Tốt nghiệp – Đại học ÞP£ Nguy n Công Nh t 1 æ4 4 4ö 3 ç + + ÷ = 1. Dấu "=" xảy ra Û x = y = z = . 16 è x y z ø 4 Baøi 6. (ĐH 2005B) Chứng minh rằng với mọi x Î R, ta có: x x x æ 12 ö æ 15 ö æ 20 ö x x x ç ÷ +ç ÷ +ç ÷ ³ 3 +4 +5 è 5ø è 4ø è 3 ø Khi nào đẳng thức xảy ra? ĐS: Áp dụng bất đẳng thức Côsi cho 2 số dương ta có: x x x æ 12 ö æ 15 ö æ 12 ö æ 15 ö ç ÷ + ç ÷ ³ 2 ç ÷ .ç ÷ è 5ø è 4ø è 5ø è 4ø Tương tự ta có: x x x x æ 12 ö æ 15 ö Þ ç ÷ + ç ÷ ³ 2.3x è 5ø è 4ø x x (1) x æ 12 ö æ 20 ö æ 15 ö æ 20 ö x x (2) (3) ç ÷ + ç ÷ ³ 2.4 ç ÷ + ç ÷ ³ 2.5 è 5ø è 3 ø è 4ø è 3 ø Cộng (1), (2), (3) và chia 2 vế của bất đẳng thức nhận được cho 2 ta có đpcm. Đẳng thức xảy ra Û (1), (2), (3) là các đẳng thức Û x = 0. Baøi 7. (ĐH 2005D) Cho các số dương x, y, z thoả mãn xyz = 1. Chứng minh rằng: 1 + x 3 + y3 1 + y 3 + z3 1 + z3 + x 3 + + ³3 3 xy yz zx Khi nào đẳng thức xảy ra? ĐS: Áp dụng bất đẳng thức Côsi cho 3 số dương ta có: 3 3 1 + x + y ³ 3 1.x .y 3 Tương tự: Mặt khác 3 1 + y3 + z3 ³ yz 3 + 3 3 = 3xy Û 3 yz 3 + 1 + x 3 + y3 ³ xy 3 3 3 Þ (1) xy 1 + z3 + x 3 ³ zx (2); ³ 33 3 3 zx 3 + (3) 3 + 3 xy yz zx xy yz zx xy yz zx Cộng các bất đẳng thức (1), (2), (3), (4) ta có đpcm. Đẳng thức xảy ra Û (1), (2), (3), (4) là các đẳng thức Û x = y = z = 1. Baøi 8. (ĐH 2005A–db1) Chứng minh rằng với mọi x, y > 0 ta có: ³3 3 (4) 2 ö æ öæ (1 + x ) ç1 + yx ÷ çç1 + 9 ÷÷ ³ 256 yø è øè Đẳng thức xảy ra khi nào? ĐS: Ta có: x x x x3 4 1+x=1+ + + ³4 3 3 3 33 1+ 1+ y y y y y3 =1+ + + ³ 44 x 3x 3x 3x 33 x 3 9 y =1+ 3 y + 3 y + 3 Trang 96 y ³ 44 33 y3 2 æ 9 ö 36 Þ ç1 + ÷ ³ 16 4 ç y ÷ø y3 è Đề thi Tốt nghiệp – Đại học 2 æ y öæ 9 ö x 3 y 3 36 Vậy: (1 + x ) ç 1 + ÷ ç 1 + . = 256 ÷ ³ 256 4 . y ÷ø 33 33 x 3 y 3 è x ø çè Baøi 9. (ĐH 2005B–db1) Cho x, y, z là 3 số dương và xyz = 1. Chứng minh rằng: x2 y2 z2 3 + + ³ 1+ y 1+ z 1+ x 2 ĐS: Ta có: x2 1 + y x2 1 + y + ³2 . =x; 1+ y 4 1+ y 4 y2 1 + z y2 1 + z + ³2 . =y 1+ z 4 1+ z 4 z2 1 + x z2 1 + x + ³2 . =z 1+ x 4 1+ x 4 Cộng 3 bất đẳng thức trên, vế theo vế, ta có: æ x 2 1 + y ö æ y 2 1 + z ö æ z2 1 + x ö + + + ç ÷+ç ÷+ç ÷³ x+y+z ç 1+ y ÷ ç 1+ z ÷ ç1+ x ÷ 4 4 4 è ø è ø è ø Û 3( x + y + z) 3 x2 y2 z2 3 x+y+z + + ³- +x+y+z ³ 4 4 1+ y 1+ z 1+ x 4 4 ³ 3 3 9 3 3 .3 - = - = (vì x + y + z ³ 3 3 xyz = 3) 4 4 4 4 2 x2 y2 z2 3 Vậy: + + ³ . 1+ y 1+ z 1+ x 2 Baøi 10. (ĐH 2005B–db2) Cho 3 số x, y, z thoả x + y + z = 0. Chứng minh rằng: 2 + 4 x + 2 + 4 y + 2 + 4z ³ 3 3 ĐS: Ta có: 3 2 + 4x = 1 + 1 + 4x ³ 3 4 x Þ 6 Vậy 2 + 4 x + 2 + 4y + 2 + 4z ³ 3 6 4x = 3 4x 6 2 + 4y ³ 3 4y ; Tương tự: 2 + 4x ³ 3 2 + 4z ³ 3 4z 3 ( 6 4 x + 6 4y + 6 4z ) ³ 3 3 36 4 x .4 y.4 z 18 ³ 3 3 4 x+ y+z = 3 3 . Baøi 11. (ĐH 2005D–db1) Cho 3 số dương a, b, c thoả mãn: a + b + c = 3 3 . Chứng minh rằng: 4 a + 3b + 3 b + 3c + 3 c + 3a £ 3 Khi nào đẳng thức xảy ra? 3 (a + 3b).1.1 £ a + 3b + 1 + 1 = 1 (a + 3b + 2) ĐS: Ta có: 3 3 3 (b + 3c).1.1 £ b + 3c + 1 + 1 = 1 (b + 3c + 2) 3 3 3 (c + 3a).1.1 £ c + 3a + 1 + 1 = 1 (c + 3a + 2) 3 3 ù 1é 3 1 Suy ra: 3 a + 3b + 3 b + 3c + 3 c + 3a £ éë 4(a + b + c) + 6 ùû £ ê 4. + 6 ú = 3 3 3ë 4 û ì 3 1 ï Dấu "=" xảy ra Û ía + b + c = 4 Ûa=b=c= 4 ïîa + 3b = b + 3c = c + 3a=1 Trang 97 Đề thi Tốt nghiệp – Đại học Nguy n Công Nh t Baøi 12. (ĐH 2005D–db2) Chứng minh rằng nếu 0 £ y £ x £ 1 thì x y - y x £ Đẳng thức xảy ra khi nào? 1 . 4 ĐS: Ta có: 0 £ x £ 1 Þ x ³ x2 1 1 x y - y x £ Û x y £ + y x (1) 4 4 Theo BĐT Côsi ta có: y x + 1 1 1 1 ³ yx 2 + ³ 2 yx 2 . = x y Þ x y - y x £ 4 4 4 4 ì ï0 £ y £ x £ 1 ì x = 1 ï ï Dấu "=" xảy ra Û í x = x 2 Ûí 1 ï 2 1 ïî y = 4 ï yx = î 4 Baøi 13. (ĐH 2006A) Cho 2 số thực x ≠ 0, y ≠ 0 thay đổi và thoả mãn điều kiện: ( x + y ) xy = x 2 + y 2 - xy Tìm giá trị lớn nhất của biểu thức: ĐS: Từ giả thiết suy ra: Đặt A= 1 x 3 + 1 y3 . 1 1 1 1 1 + = + - . x y x 2 y 2 xy 1 1 = a , = b , ta có: a + b = a2 + b2 - ab x y (1) A = a3 + b3 = (a + b)(a2 - ab + b2 ) = (a + b)2 Từ (1) suy ra: a + b = (a + b)2 - 3ab . 2 æa+bö 2 3 2 Vì ab £ ç ÷ nên a + b ³ (a + b) - (a + b) 4 è 2 ø 2 Þ (a + b) – 4(a + b) ≤ 0 Þ 0 ≤ a + b ≤ 4 Suy ra: A = (a + b)2 ≤ 16 1 Với x = y = thì A = 16. Vậy giá trị lớn nhất của A là 16. 2 Baøi 14. (ĐH 2006B) Cho x, y là các số thực thay đổi. Tìm giá trị nhỏ nhất của biểu thức: A = ( x - 1)2 + y 2 + ( x + 1)2 + y 2 + y - 2 ĐS: Trong mpOxy, xét hai điểm M ( x - 1; - y ), N ( x + 1; y ) . Do OM + ON ≥ MN nên: ( x - 1) 2 + y2 + ( x + 1) 2 + y2 ³ 4 + 4y2 = 2 1 + y2 Do đó: A ≥ 2 1 + y 2 + y - 2 = f(y) · Với y ≤ 2 Þ f(y) = 2 1+ y 2 + 2 – y Þ f ¢(y) = 2y y2 + 1 ìy ³ 0 1 f ¢(y) = 0 Û 2y = 1+ y 2 Û í 2 2 Ûy= 3 î4 y = 1 + y Do đó ta có bảng biến thiên như trên · Với y ≥ 2 Þ f(y) ≥ 2 1+ y 2 ≥ 2 5 > 2 + Trang 98 3. –1 Đề thi Tốt nghiệp – Đại học 3 với mọi số thực x, y. 1 Khi x = 0 và y = thì A = 2 + 3 . Nên giá trị nhỏ nhất của A là 2 + 3 Vậy A ≥ 2 + 3. Baøi 15. (ĐH 2006A–db1) Cho x, y là các số thực thoả mãn điều kiện: x 2 + xy + y 2 £ 3 . Chứng minh rằng: -4 3 - 3 £ x 2 - xy - 3y 2 £ 4 3 - 3 . ĐS: Đặt A = x 2 + xy + y 2 , B = x 2 - xy - 3y 2 . · Nếu y = 0 thì A = x 2 £ 3 Þ B = x 2 . Do đó: -4 3 - 3 £ 0 £ B £ 3 < 4 3 - 3 Þ đpcm x A( x 2 - xy - 3y 2 ) t2 - t - 3 · Nếu y ¹ 0. Đặt t = . Ta có B = . =A y x 2 + xy + y 2 t2 + t + 1 Ta tìm tập giá trị của u = t2 - t - 3 2 Û (u - 1)t 2 + (u + 1)t + u + 3 = 0 . t + t +1 Vì a = u - 1, b = u + 1 không đồng thời bằng 0 nên miện giá trị của u là: D³0Û -4 3 - 3 4 3 -3 £u£ 3 3 Ta có: B = A.u và 0 £ A £ 3 Þ -4 3 - 3 £ B £ 4 3 - 3 . Baøi 16. (ĐH 2006A–db2) Cho các số thực x, y, z thỏa mãn: 3– x + 3–y + 3–z = 1 . Chứng minh rằng: 9x 9y 9z 3x + 3 y + 3z + + ³ 4 3 x + 3 y+ z 3 y + 3 z+ x 3 z + 3 x+ y ĐS: Đặt 3 x = a , 3y = b , 3z = c . Ta có a, b, c > 0 và ab + bc + ca = abc . a2 b2 c2 a+b+c BĐT Û + + ³ a + bc b + ca c + ab 4 3 3 3 a b c a+b+c Û 2 + 2 + 2 ³ 4 a + abc b + abc c + abc 3 3 a b c3 a+b+c Û + + ³ (1) (a + b)(a + c) (b + c)(b + a) (c + a )(c + b) 4 Áp dụng BĐT Cô–si ta có: a3 a+b a+c a3 a+b a+c 3 + + ³ 33 × × = a (a + b)(a + c) 8 8 (a + b)(a + c) 8 8 4 (2) b3 b+c b+a b3 b+c b+a 3 + + ³ 33 × × = b (b + c)(b + a ) 8 8 (b + c)(b + a ) 8 8 4 (3) c3 c+a c+b c3 c+a c+b 3 + + ³ 33 × × = c (4) (c + a )(c + b) 8 8 (c + a )(c + b) 8 8 4 Cộng vế theo vế các bất đẳng thức (2), (3), (4) ta suy ra: a3 b3 c3 a+b+c + + ³ (a + b)(a + c) (b + c)(b + a) (c + a )(c + b) 4 Vậy (1) đúng và ta có điều phải chứng minh. Baøi 17. (ĐH 2006B–db2) Cho hai số dương x, y thay đổi thoả mãn điều kiện: x + y ³ 4 . Tìm giá trị nhỏ nhất của biểu thức: A= 3 x 2 + 4 2 + y3 . + 4x y2 Trang 99 Đề thi Tốt nghiệp – Đại học ĐS: Ta có: A = Nguy n Công Nh t æ 1 y yö x+y x 1 1 9 + + 2ç + + ÷ + ³ 1 + 2.3. + 2 = . 2 4 x 8 8ø 2 4 2 èy ìx 1 ïï 4 = x 9 Dấu "=" xảy ra Û í 1 y Û x = y = 2 (thoả x + y ³ 4). Vậy minA = . 2 ï = 2 8 ïî y Baøi 18. (ĐH 2007A) Cho x, y, z là các số thực dương thay đổi và thoả mãn điều kiện xyz = 1 . Tìm giá trị nhỏ nhất của biểu thức: x 2 ( y + z) P= y y + 2z z + y2 (z + x ) z z + 2x x + z2 ( x + y ) x x + 2y y . ĐS: Ta có: x 2 ( y + z) = x 2 y + x 2 z ³ 2 x 3 xyz = 2 x x . Tương tự: y 2 ( z + x ) ³ 2 y y ; z2 ( x + y ) ³ 2 z z . ÞP³ 2x x y y + 2z z + 2y y z z + 2x x + 2z z x x + 2y y . Đặt a = x x + 2 y y , b = y y + 2 z z , c = z z + 2 x x . 4c + a - 2 b 4 a + b - 2c 4b + c - 2a ,y y= ,z z= . 9 9 9 2 æ 4c + a - 2b 4a + b - 2c 4b + c - 2a ö Do đó: P ³ ç + + ÷ 9è b c a ø 2é æc a bö ù 2 c a b c a b = ê5 ç + + ÷ - 6 ú ³ (5.3 - 6) = 2 (do + + ³ 3 3 . . = 3 ) 9ë èb c aø û 9 b c a b c a dấu "=" xảy ra Û x = y = z =1. Vậy minP = 2. Baøi 19. (ĐH 2007B) Cho x, y, z là ba số thực dương thay đổi. Tìm giá trị nhỏ nhất của biểu æx 1 ö æy 1 ö æz 1 ö thức: P = x ç + ÷ + yç + ÷ + zç + ÷ . è 2 zx ø è 2 xy ø è 2 yz ø Þ x x= ĐS: Ta có: P = Do nên x 2 y 2 z 2 x 2 + y 2 + z2 + + + . 2 2 2 xyz x 2 + y 2 y 2 + z2 z2 + x 2 x +y +z = + + ³ xy + yz + zx 2 2 2 æ x 2 1 ö æ y 2 1 ö æ z2 1 ö P ³ ç + ÷+ç + ÷+ç + ÷ è 2 xø è 2 yø è 2 zø 2 2 2 t2 1 + với t > 0. Lập bảng biến thiên của f(t), ta suy ra: 2 t 3 9 f (t ) ³ , "t > 0 . Suy ra: P ³ . Dấu "=" xảy ra Û x = y = z = 1 2 2 9 Vậy minP = . 2 Xét hàm số: f (t ) = b a æ 1 ö æ b 1 ö Baøi 20. (ĐH 2007D) Cho a ³ b > 0 . Chứng minh rằng: ç 2a + ÷ £ ç2 + b ÷ . è 2a ø è 2 ø Trang 100 Đề thi Tốt nghiệp – Đại học ln(1 + 4 a ) ln(1 + 4 b ) £ . a b ĐS: BĐT Û (1 + 4 a )b £ (1 + 4b )a Û Xét hàm số: f ( x ) = ln(1 + 4 x ) 4 x ln 4 x - (1 + 4 x ) ln(1 + 4 x ) với x > 0. Ta có: f ¢ ( x ) = 0 ta có f(a) £ f(b) Þ đpcm. Baøi 21. (ĐH 2007A–db2) Cho x, y, z là các số dương. Tìm giá trị nhỏ nhất của biểu thức: æ x y z ö P = 3 4( x 3 + y 3 ) + 3 4( y 3 + z3 ) + 3 4( z3 + x 3 ) + 2 ç + + ÷ . 2 2 z x2 ø èy ĐS: Với x, y ,z > 0, ta chứng minh được: 4( x 3 + y 3 ) ³ ( x + y )3 . Dấu "=" xảy ra Û x = y. Tương tự: 4( y3 + z3 ) ³ ( y + z)3 , 4( z3 + x 3 ) ³ (z + x )3 . Do đó: 3 4( x 3 + y 3 ) + 3 4( y3 + z3 ) + 3 4( z3 + x 3 ) ³ 2( x + y + z) ³ 6 3 xyz æ x y z ö 6 Mặt khác: 2 ç + + ÷ ³ . Dấu "=" xảy ra Û x = y = z. 2 2 2 3 xyz y z x è ø æ 1 ö Vậy: P ³ 6 ç 3 xyz + ÷ ³ 12 . Dấu "=" xảy ra Û x = y = z = 1. Vậy minP = 12. ç 3 xyz ÷ è ø Baøi 22. (ĐH 2007D–db1) Cho a, b là các số dương thoả mãn ab + a + b = 3 . Chứng minh: 3a 3b ab 3 + + £ a2 + b2 + . b +1 a +1 a + b 2 ĐS: Từ giả thiết Þ ab = 3 - (a + b), (a + 1)(b + 1) = ab + a + b + 1 = 4 . BĐT Û a2 + b2 + 3 3a(a + 1) + 3b(b + 1) 3 ³ + -1 2 (a + 1)(b + 1) a+b Û a2 + b2 - 3(a + b) - 12 + 10 ³ 0 (*) a+b Đặt x = a + b Þ x 2 = (a + b)2 ³ 4ab = 4(3 - x ) Þ x 2 + 4 x - 12 ³ 0 Þ x ³ 2 (vì x > 0). Ta có (*) Û x 2 - x - 12 + 4 ³ 0 với x ³ 2. x Û ( x - 2)( x 2 + x + 6) ³ 0 với x ³ 2 (hiển nhiên đúng). Baøi 23. (ĐH 2008B) Cho hai số thực x, y thay đổi và thoả mãn hệ thức x 2 + y 2 = 1 . Tìm giá trị lớn nhất và giá trị nhỏ nhất của biểu thức P = ĐS: Ta có: P = 2( x 2 + 6 xy ) x 2 + y 2 + 2 xy + 2 y 2 2( x 2 + 6 xy ) 1 + 2 xy + 2 y 2 . . · Nếu y = 0 thì x 2 = 1 Þ P = 2. · Nếu y ¹ 0. Đặt x = ty , khi đó: P = 2t 2 + 12t 2 Û (P - 2)t 2 + 2(P - 6)t + 3P = 0 (1) t + 2t + 3 3 – Với P = 2, phương trình (1) có nghiệm t = . 4 – Với P ¹ 2, phương trình (1) có nghiệm Û D¢ = -2P 2 - 6 P + 36 ³ 0 Û -6 £ P £ 3 . 3 1 3 1 + P = 3 khi x = ,y= hoặc x = , y=. 10 10 10 10 Trang 101 Đề thi Tốt nghiệp – Đại học + P = –6 khi x = 3 Nguy n Công Nh t 2 , y=- hoặc x = - 3 2 ,y= . 13 13 13 13 Vậy: maxP = 3, minP = –6. Baøi 24. (ĐH 2008D) Cho x, y là hai số thực không âm thay đổi. Tìm giá trị lớn nhất và giá trị ( x - y)(1 - xy ) nhỏ nhất của biểu thức: P = . (1 + x )2 (1 + y )2 ĐS: Ta có: P = ( x - y )(1 - xy ) 2 2 (1 + x ) (1 + y ) £ ( x + y )(1 + xy ) 2 [( x + y) + (1 + xy)] £ 1 1 1 Û - £P£ . 4 4 4 1 1 · Khi x = 0, y = 1 thì P = - . · Khi x = 1, y = 0 thì P = . 4 4 1 1 Vậy: minP = - , maxP = . 4 4 Baøi 25. (ĐH 2009A) Chứng minh rằng với mọi số thực dương x, y, z thoả mãn điều kiện: x( x + y + z) = 3yz , ta có: ( x + y )3 + ( x + z)3 + 3( x + y )( x + z)( y + z) £ 5( y + z)3 . ĐS: Đặt a = x + y, b = x + z, c = y + z . 3 1 Điều kiện x( x + y + z) = 3yz Û c2 = a2 + b2 - ab = (a + b)2 - (a + b)2 = (a + b)2 4 4 Þ a + b £ 2c (1) BĐT Û a3 + b3 + 3abc £ 5c3 Û (a + b)(a2 + b2 - ab) + 3abc £ 5c3 Û (a + b)c2 + 3abc £ 5c3 Û (a + b)c + 3ab £ 5c3 3 (a + b)2 £ 3c 2 Þ đpcm. 4 Dấu "=" xảy ra Û a = b = c Û x = y = z. (1) cho ta: (a + b)c £ 2c2 và 3ab £ Baøi 26. (ĐH 2009B) Cho các số thực x, y thay đổi và thoả mãn ( x + y )3 + 4 xy ³ 2 . Tìm giá trị nhỏ nhất của biểu thức: A = 3( x 4 + y 4 + x 2 y 2 ) - 2( x 2 + y 2 ) + 1 . ĐS: Từ ( x + y)3 + 4 xy ³ 2 với ( x + y)2 ³ 4 xy Þ ( x + y )3 + ( x + y )2 ³ 2 Þ x + y ³ 1 . 3 2 3 ( x + y 2 )2 + ( x 4 + y 4 ) - 2( x 2 + y 2 ) + 1 2 2 3 3 9 ³ ( x 2 + y 2 )2 + ( x 2 + y 2 )2 - 2( x 2 + y 2 ) + 1 Þ A ³ ( x 2 + y 2 )2 - 2( x 2 + y 2 ) + 1 2 4 4 A = 3( x 4 + y 4 + x 2 y 2 ) - 2( x 2 + y 2 ) + 1 = ( x + y )2 1 1 9 Đặt t = x + y , ta có: x + y ³ ³ Þ t ³ . Do đó A ³ t 2 - 2t + 1 . 2 2 2 4 æ1ö 9 9 9 1 Xét f (t ) = t 2 - 2t + 1 ; f ¢ (t ) = t - 2 > 0 , "t ³ Þ min f (t ) = f ç ÷ = . é1 ö 4 2 2 è 2 ø 16 ;+¥ 2 2 2 2 ê2 ë ÷ ø 9 1 9 . Dấu "=" xảy ra khi x = y = . Vậy minA = . 16 2 16 Baøi 27. (ĐH 2009D) Cho các số thực không âm x, y thay đổi và thoả mãn x + y = 1 . Tìm giá Þ A³ trị nhỏ nhất và giá trị lớn nhất của biểu thức S = (4 x 2 + 3 y )(4 y 2 + 3 x ) + 25 xy . ĐS: Do x + y = 1 nên S = 16 x 2 y 2 + 12( x 3 + y 3 ) + 9 xy + 25 xy = S = 16 x 2 y 2 + 12 éë( x + y )3 - 3xy( x + y )ùû + 34 xy = 16 x 2 y 2 - 2 xy + 12 Trang 102 Đề thi Tốt nghiệp – Đại học é 1ù Xét hàm số: f (t ) = 16t 2 - 2t + 12 trên đoạn ê 0; ú ë 4û æ 1 ö 191 æ 1 ö 25 1 f ¢ (t ) = 32t - 2; f ¢ (t ) = 0 Û t = ; f (0) = 12, f ç ÷ = , fç ÷= . 16 è 16 ø 16 è4ø 2 æ 1 ö 25 æ 1 ö 191 Þ max f (t ) = f ç ÷ = ; min f (t ) = f ç ÷ = é 1ù è 4 ø 2 é 0; 1 ù è 16 ø 16 0; êë 4 úû êë 4 úû ìx + y = 1 æ1 1ö 25 ï Giá trị lớn nhất của S bằng khi í 1 Û ( x; y ) = ç ; ÷ . 2 è2 2ø ïî xy = 4 ìx + y = 1 æ2+ 3 2- 3 ö 191 ï Giá trị nhỏ nhất của S bằng khi í ; ÷ 1 Û ( x; y ) = ç 16 è 4 4 ø ïî xy = 16 æ2- 3 2+ 3 ö hoặc ( x; y ) = ç ; ÷ è 4 4 ø Baøi 28. (ĐH 2010B) Cho các số thực không âm a, b, c thoả mãn: a + b + c = 1 . Tìm giá trị nhỏ nhất của biểu thức M = 3(a2 b2 + b2 c2 + c 2 a2 ) + 3(ab + bc + ca) + 2 a2 + b2 + c 2 . ĐS: Ta có: M ³ (ab + bc + ca)2 + 3(ab + bc + ca) + 2 1 - 2(ab + bc + ca) . (a + b + c)2 1 = . 3 3 é 1ö 2 Xét hàm số: f (t ) = t 2 + 3t + 2 1 - 2t trên ê 0; ÷ . Ta có: f ¢ (t ) = 2t + 3 . 1 - 2t ë 2ø 2 f ¢¢ (t ) = 2 £ 0 , dấu "=" chỉ xảy ra tại t = 0 Þ f ¢(t) nghịch biến. (1 - 2t )3 Đặt t = ab + bc + ca , ta có: 0 £ t £ é 1ù æ 1 ö 11 Xét trên đoạn ê 0; ú ta có: f ¢ (t ) ³ f ç ÷ = - 2 3 > 0 , suy ra f(t) nghịch biến. ë 3û è3ø 3 é 1ù Do đó: f(t) ³ f(0) = 2, "t Î ê 0; ú . ë 3û é 1ù Vì thế: M ³ f (t ) ³ 2, "t Î ê 0; ú ; ë 3û M = 2 khi ab = bc = ca, ab + bc + ca = 0 và a + b + c = 1 Û (a; b; c) là một trong các bộ số (1; 0; 0), (0; 1; 0), (0; 0; 1). Do đó: minM = 2. Baøi 29. (ĐH 2010D) Tìm giá trị nhỏ nhất của hàm số y = - x 2 + 4 x + 21 - - x 2 + 3 x + 10 . ĐS: Điều kiện: –2 £ x £ 5. Ta có: (- x 2 + 4 x + 21) - (- x 2 + 3 x + 10) = x + 11 > 0 Þ y > 0 . y 2 = ( x + 3)(7 - x ) + ( x + 2)(5 - x ) - 2 ( x + 3)(7 - x )( x + 2)(5 - x ) = ( ( x + 3)(5 - x ) - ( x + 2)(7 - x ) ) 2 +2³2 1 . Vậy miny = 2 . 3 Baøi 30. (ĐH 2011A) Cho x, y, z là ba số thực thuộc đoạn [1; 4] và x ³ y, x ³ z. Tìm giá trị nhỏ Suy ra: y ³ 2 . Dấu "=" xảy ra Û x = Trang 103 Đề thi Tốt nghiệp – Đại học Nguy n Công Nh t P= nhất của biểu thức: x y z + + . 2 x + 3y y + z z + x 1 1 2 + ³ (*), với a, b > 0 và ab ³ 1. 1 + a 1 + b 1 + ab Dấu "=" xảy ra Û a = b hoặc ab = 1. Áp dụng (*) với x, y Î [1; 4] và x ³ y, ta có: x 1 1 1 2 P= + + ³ + z x 3y 2 x + 3y x 1+ 1+ 2+ 1+ y z x y ĐS: Trước hết chứng minh Dấu "=" xảy ra Û Đặt z x x = hoặc = 1 y z y (1) x t2 2 = t, t Î [1;2] . Khi đó: P ³ + . y 2t 2 + 3 1 + t t2 2 , t Î [1;2] . 2t + 3 1 + t 34 x Ta có: f (t ) ³ f (2) = ; dấu "=" xảy ra Û t = 2 Û = 4 Û x = 4, y = 1 33 y Xét hàm f (t ) = + 2 (2) 34 . từ (1) và (2) suy ra dấu "=" xảy ra Û x = 4, y = 1, z = 2 . 33 34 Vậy minP = khi x = 4, y = 1, z = 2. 33 Þ P³ Baøi 31. (ĐH 2011B) Cho a, b là các số thực dương thoả mãn 2(a2 + b2 ) + ab = (a + b)(ab + 2) . æ a3 b3 ö æ a 2 b 2 ö Tìm giá trị nhỏ nhất của biểu thức P = 4 çç + ÷÷ - 9 çç + ÷÷ . è b3 a3 ø è b 2 a 2 ø ĐS: Với a, b > 0 ta có: æa bö æ1 1ö 2(a2 + b2 ) + ab = (a + b)(ab + 2) Û 2 ç + ÷ + 1 = (a + b) + 2 ç + ÷ . èb aø èa bø æ1 1ö æ1 1ö æa b ö Mà (a + b) + 2 ç + ÷ ³ 2 2(a + b) ç + ÷ = 2 2 ç + + 2 ÷ èa bø èa bø èb a ø æa bö æa b ö a b 5 Suy ra: 2 ç + ÷ + 1 ³ 2 2 ç + + 2 ÷ Þ + ³ . b a 2 èb aø èb a ø a b 5 Đặt t = + , t ³ , suy ra: P = 4(t 3 - 3t ) - 9(t 2 - 2) = 4t 3 - 9t 2 - 12t + 18 . b a 2 5 Xét hàm số f (t ) = 4t 3 - 9t 2 - 12t + 18, t ³ . 2 æ5ö 23 Ta suy ra được min f (t ) = f ç ÷ = - . é5 ö è2ø 4 ;+¥ ê2 ë ÷ ø ìa b 5 + = ï 23 b a 2 Vậy minP = Ûí Û (a; b) = (1; 2) hoặc (a; b) = (2; 1). 4 ïa + b = 2 æç 1 + 1 ö÷ èa bø îï Trang 104 Đề thi Tốt nghiệp – Đại học Baøi 32. (ĐH 2012A) Cho các số thực x , y, z thoả mãn điều kiện x + y + z = 0 . Tìm giá trị nhỏ P = 3 x - y + 3 y - z + 3 z - x - 6 x 2 + 6 y 2 + 6 z2 . nhất của biểu thức: ĐS: Trước hết chứng minh 3t ³ t + 1, "t ³ 0 (*) (xét hàm f (t ) = 3t - t - 1, t ³ 0 ). Áp dụng (*), ta có: 3 x - y + 3 y - z + 3 z - x ³ 3 + x - y + y - z + z - x . Áp dụng BĐT a + b ³ a + b , ta có: 2 ( x - y + y -z + z- x ) = x - y2 + y -z2 + z- x 2 + x - y ( y-z + z- x ) + 2 2 2 + y - z ( z - x + x - y ) + z - x ( x - y + y - z ) ≥ 2( x - y + y - z + z - x ) Do đó: ( 2 2 x-y + y-z + z-x ³ 2 x-y + y-z + z-x 2 )= 6( x 2 + y 2 + z2 ) - 2( x + y + z)2 Mà x + y + z = 0 , suy ra x - y + y - z + z - x ³ 6( x 2 + y 2 + z2 ) . Suy ra P = 3 x - y + 3 y - z + 3 z - x - 6 x 2 + 6 y 2 + 6 z2 ³ 3 . Dấu "=" xảy ra Û x = y = z = 0 . Vậy min P = 3 khi x = y = z = 0 . Baøi 33. (ĐH 2012B) Cho các số thực x , y, z thoả mãn các điều kiện x + y + z = 0 và x 2 + y 2 + z2 = 1 . Tìm giá trị lớn nhất của biểu thức P = x 5 + y 5 + z5 . ĐS: Với x + y + z = 0 và x 2 + y 2 + z2 = 1 , ta có: 0 = ( x + y + z)2 = x 2 + y 2 + z2 + 2 x ( y + z) + 2 yz = 1 - 2 x 2 + 2 yz , nên yz = x 2 Mặt khác yz £ y 2 + z2 1 - x 2 1 1 - x2 6 6 = , suy ra x 2 - £ , do đó £x£ 2 2 2 2 3 3 1 . 2 (*) Khi đó: P = x 5 + ( y 2 + z2 )( y3 + z3 ) - y 2 z2 ( y + z) 2 æ 1ö 5 = x + (1 - x ) éë( y 2 + z2 )( y + z) - yz( y + z)ùû + ç x 2 - ÷ x = (2 x 3 - x ) . è 2ø 4 é 6 6ù 6 Xét hàm số f ( x ) = 2 x 3 - x trên D = ê ; , "x Î D . ú . Ta có f ( x ) £ 9 ë 3 3 û 5 Suy ra P £ 2 5 6 6 6 5 6 . Dấu "=" xảy ra Û x = ,y = z = . Vậy max P = . 36 3 6 36 Baøi 34. (ĐH 2012D) Cho các số thực x , y thoả mãn ( x - 4)2 + ( y - 4)2 + 2 xy £ 32 . Tìm giá trị nhỏ nhất của biểu thức A = x 3 + y 3 + 3( xy - 1)( x + y - 2) . ĐS: Ta có: ( x - 4)2 + ( y - 4)2 + 2 xy £ 32 Û ( x + y )2 - 8( x + y ) £ 0 Û 0 £ x + y £ 8 3 A = ( x + y)3 - 3( x + y) - 6 xy + 6 ³ ( x + y )3 - ( x + y )2 - 3( x + y ) + 6 . 2 17 - 5 5 3 Xét hàm số f (t ) = t 3 - t 2 - 3t + 6 trên đoạn [0;8], ta có min f (t ) = . 2 4 Suy ra A ³ 17 - 5 5 1+ 5 17 - 5 5 . Dấu "=" xảy ra Û x = y = . Vậy min A = . 4 4 4 Baøi 35. (ĐH 2013A) Cho các số thực dương a, b, c thoả mãn điều kiện (a + c)(b + c) = 4c2 . Trang 105 Đề thi Tốt nghiệp – Đại học Nguy n Công Nh t P= Tìm giá trị nhỏ nhất của biểu thức 32a3 + (b + 3c)3 32b3 (a + 3c)3 - a2 + b2 . c a b ĐS: Đặt x = , y = ( x > 0, y > 0) . ĐK bài toán trở thành: xy + x + y = 3 . c c Khi đó: P = 32 x 3 3 ( y + 3) + 32 y 3 3 ( x + 3) - x 2 + y2 . 3 (u + v)3 Với "u > 0, v > 0 ta có: u3 + v3 = (u + v)3 - 3uv(u + v) ³ (u + v)3 - (u + v)3 = . 4 4 3 3 æ ( x + y )2 - 2 xy + 3 x + 3y ö æ x y ö Do đó: + ³ 8ç + = 8 ç ÷ ÷ xy + 3 x + 3y + 9 ( y + 3)3 ( x + 3)3 è y+3 x +3ø è ø 32 x 3 32 y3 32 x 3 3 32 y 3 æ ( x + y - 1)( x + y + 6) ö 3 Thay xy + x + y = 3 ta được: + ³ 8ç ÷ = ( x + y - 1) . 3 3 2( x + y + 6) ( y + 3) ( x + 3) è ø Do đó: P ³ ( x + y - 1)3 - x 2 + y 2 = ( x + y - 1)3 - ( x + y )2 + 2( x + y ) - 6 . Đặt t = x + y, t > 0 ta được: P ³ (t - 1)3 - t 2 + 2t - 6 . Ta có: 3 = x + y + xy £ ( x + y ) + ( x + y )2 t2 =t+ nên (t - 2)(t + 6) ³ 0 Þ t ³ 2 . 4 4 Xét f (t ) = (t - 1)3 - t 2 + 2t - 6, t ³ 2 Þ f ¢(t ) ³ 3 - 3 2 > 0 Þ t(t ) ³ f (2) = 1 - 2 . 2 Do đó P ³ 1 - 2 . Khi a = b = c thì P = 1 - 2 . Do đó min P = 1 - 2 . Baøi 36. (ĐH 2013B) Cho a, b, c là các số thực dương. Tìm giá trị lớn nhất của biểu thức: P= 4 2 2 2 a +b +c +4 - 9 (a + b) (a + 2c)(b + 2c) . ĐS: Ta có: a + b + 4c a2 + b2 + 2ab + 4ac + 4bc = £ 2(a2 + b2 + c 2 ) . 2 2 4 9 4 9 Đặt t = a2 + b2 + c2 + 4, t > 2 Þ P £ . Xét f (t ) = ,t > 2 . t 2(t 2 - 4) t 2(t 2 - 4) (a + b) (a + 2c)(b + 2c) £ (a + b) 5 5 Þ f ¢(t ) > 0, "t > 2 . Dựa vào BBT Þ f (t ) £ , "t > 2 Þ P £ . 8 8 5 5 Khi a = b = c = 2 ta có P = . Vậy max P = . 8 8 Baøi 37. (ĐH 2013D) Cho x, y là các số thực dương thoả mãn điều kiện xy £ y - 1 . Tìm giá trị lớn nhất của biểu thức P= x+y x 2 - xy + 3y 2 - x - 2y . 6( x + y ) 2 x y -1 1 1 1 æ 1 1 ö 1 ĐS: Do x > 0, y > 0, xy £ y - 1 nên 0 < £ = = -ç - ÷ £ . 2 2 y y y 4 èy 2ø 4 y Đặt t = x 1 t +1 t -2 Þ 0 < t £ . Khi đó P = . y 4 t 2 - t + 3 6(t + 1) Trang 106 Đề thi Tốt nghiệp – Đại học Xét f (t ) = t +1 t2 - t + 3 - t -2 1 1 1 ,0 < t £ Þ f ¢(t ) > - >0. 6(t + 1) 4 3 2 æ1ö 5 7 Do đó P = f (t ) £ f ç ÷ = + . è 4 ø 3 30 1 5 7 5 7 Khi x = , y = 2 ta có P = + . Do đó max P = + . 2 3 30 3 30 Baøi 38. (ĐH 2014A) Cho x, y, z là các số thực không âm thoả mãn điều kiện x 2 + y 2 + z2 = 2 . P= Tìm giá trị lớn nhất của biểu thức x2 x 2 + yz + x + 1 + y+z 1 + yz . x + y + z +1 9 ĐS: Ta có: 0 £ ( x - y - z)2 = x 2 + y 2 + z2 - 2 xy - 2 xz + 2 yz = 2(1 - xy - xz + yz) Nên x 2 + yz + x + 1 = x ( x + y + z + 1) + (1 - xy - xz + yz) ³ x ( x + y + z + 1) Suy ra x2 2 x + yz + x + 1 £ x . x + y + z =1 Mặt khác ( x + y + z)2 = x 2 + y 2 + z2 + 2 x ( y + z) + 2 yz = 2 + 2 yz + 2 x ( y + z) £ x+y+z ( x + y + z)2 £ 2 + 2 yz + éë x 2 + ( y + z)2 ùû = 4(1 + yz) . Do đó P £ . x + y + z +1 36 Đặt t = x + y + z, t ³ 0 Þ t 2 = ( x + y + z)2 = ( x 2 + y 2 + z2 ) + 2 xy + 2 yz + 2 xz £ £ 2 + ( x 2 + y 2 ) + ( y 2 + z2 ) + (z2 + x 2 ) = 6 . Do đó 0 £ t £ 6 . t t2 5 5 Xét f (t ) = - , 0 £ t £ 6 . Ta được f (t ) £ khi 0 £ t £ 6 . Do đó P £ . t + 1 36 9 9 5 5 Khi x = y = 1, z = 0 thì P = . Do đó max P = . 9 9 Baøi 39. (ĐH 2014B) Cho các số thực a, b, c không âm và thoả mãn điều kiện (a + b)c > 0 . P= Tìm giá trị nhỏ nhất của biểu thức a b c + + . b+c a + c 2(a + b) a 2a ³ . b+c a+b+c ĐS: Ta có a + b + c ³ 2 a(b + c) . Suy ra b 2b ³ . a+c a+b+c é 2(a + b) a + b + c ù 1 2(a + b) c 1 3 Do đó P ³ + =ê + - ³ 2- = . ú a + b + c 2(a + b) ë a + b + c 2(a + b) û 2 2 2 Tương tự: 3 3 . Do đó min P = . 2 2 Baøi 40. (ĐH 2014D) Cho 2 số thực x, y thoả mãn các điều kiện 1 £ x £ 2;1 £ y £ 2 . Tìm giá trị Khi a = 0, b = c > 0 thì P = nhỏ nhất của biểu thức P= x + 2y x 2 + 3y + 5 + y + 2x y 2 + 3x + 5 + 1 . 4( x + y - 1) ĐS: Do 1 £ x £ 2 nên ( x - 1)( x - 2) £ 0 Û x 2 + 2 £ 3 x . Tương tự y 2 + 2 £ 3y . Suy ra P ³ x + 2y y + 2x 1 x+y 1 + + = + . 3 x + 3y + 3 3y + 3 x + 3 4( x + y - 1) x + y + 1 4( x + y - 1) Trang 107 Đề thi Tốt nghiệp – Đại học Đặt t = x + y Þ 2 £ t £ 4 . Xét f (t ) = Do đó P ³ Nguy n Công Nh t t 1 7 + , 2 £ t £ 4 . Suy ra f (t ) ³ f (3) = . t + 1 4(t - 1) 8 7 7 7 . Khi x = 1, y = 2 thì P = . Vậy min P = . 8 8 8 Baøi 41. (ĐH 2015A) ĐS: Baøi 42. (ĐH 2015B) ĐS: Baøi 43. (ĐH 2015D) ĐS: Baøi 44. (CĐ 2015) ĐS: Trang 108 Đề thi Tốt nghiệp – Đại học Phụ lục III. LƯỢNG GIÁC ĐỀ THI ĐẠI HỌC Baøi 1. (ĐH 2002A–db2) Gọi A, B, C là ba góc của tam giác ABC. Chứng minh rằng để tam giác ABC đều thì điều kiện cần và đủ là: A B C 1 A-B B -C C-A cos2 + cos2 + cos2 - 2 = cos cos cos (*) 2 2 2 4 2 2 2 A-B B -C C-A ĐS: (*) Û 2(3 + cos A + cos B + cos C ) - 8 = cos cos cos 2 2 2 A-B B -C C-A Û 2(cos A + cos B + cos C - 1) = cos cos cos 2 2 2 A B C A-B B -C C-A Û 8sin sin sin = cos cos cos 2 2 2 2 2 2 Û 8sin A.sin B.sin C = (sin A + sin B)(sin B + sin C )(sin C + sin A) Û sin A = sin B = sin C (dùng BĐT Cô–si cho vế phải) Û A = B = C. Baøi 2. (ĐH 2002B–db2) Cho tam giác ABC có độ dài các cạnh là BC = a, CA = b, AB = c. Tính diện tích tam giác ABC, biết rằng: b sin C (b.cos C + c.cos B) = 20 (*). ĐS: (*) Û 4 R 2 sin B sin C (sin B cos C + sin C cos B ) = 20 (dùng định lí hàm số sin) Û 4 R 2 sin A sin B sin C = 20 abc 8R3 = sin A sin B sin C = 2 R 2 sin A sin B sin C = 10. 4R 4R Vậy S = 10 (đvdt). Baøi 3. (ĐH 2003A–db1) Tính các góc của tam giác ABC, biết rằng: ì4 p( p - a) £ bc (1) ï í A B C 2 3 -3 (2) ïîsin 2 sin 2 sin 2 = 8 a+b+c trong đó BC = a, CA = b, AB = c, p = . 2 Mà S = (a + b + c)(b + c - a) (b + c)2 - a2 2bc(1 + cos A) £1Û £1 Û £1 bc bc bc æ A 3 1 A 3 A pö Û cos2 £ Û sin2 ³ Û sin ³ (3) ç do 0 < < ÷ . 4 2 4 2 2 è 2 2ø Biến đổi vế trái của (2): A B C 1 Aæ B -C B +C ö 1 Aæ Aö sin sin sin = sin ç cos - cos ÷ £ sin ç 1 - sin ÷ = 2 2 2 2 2è 2 2 ø 2 2è 2ø 2 1 2A 1 A 1é 2 A Aù 1 éêæ A 1 ö 1 ùú = - sin + sin = - êsin - sin ú = - ç sin - ÷ 2 2 2 2 2ë 2 2û 2 ëêè 2 2 ø 4 ûú ĐS: (1) Û 2 1 1æ A 1ö = - ç sin - ÷ . 8 2è 2 2ø 2 A B C 1 1æ 3 1ö 2 3 -3 Do (3) suy ra sin sin sin £ - ç - ÷ = . 2 2 2 8 2è 2 2ø 8 Trang 109 Đề thi Tốt nghiệp – Đại học Nguy n Công Nh t ì B -C ïïcos 2 = 1 ìï A = 1200 Dấu "=" xảy ra Û í Ûí . 0 ïî B = C = 30 ïsin A = 3 ïî 2 2 Baøi 4. (ĐH 2003D–db1) Tìm các góc A, B, C của tam giác ABC để biểu thức sau đạt giá trị nhỏ nhất: Q = sin2 A + sin 2 B - sin2 C . 1 1 ĐS: Ta có: Q = (1 - cos 2 A) + (1 - cos 2B) - sin2 C 2 2 1 = 1 - .2 cos( A + B).cos( A - B) - sin2 C = cos2 C + cos C.cos( A - B) 2 2 é ù 1 1 1 = ê cos C + cos( A - B)ú - cos2 ( A - B) ³ ë 2 û 4 4 ìA = B 1 1 ï ïìC = 120 0 Vậy Q ³ - và Q = - Û í . 1Ûí 0 cos C = 4 4 A = B = 30 ï ïî î 2 Baøi 5. (ĐH 2003D–db2) Xác định dạng của tam giác ABC, biết rằng: ( p - a)sin2 A + ( p - b)sin 2 B = c.sin A.sin B (1) a+b+c . 2 ( p - a)a ( p - b)b ĐS: (1) Û ( p - a)a2 + ( p - b)b2 = abc Û + =1 bc ac A p( p - a) p( p - b) B Û a+ b = p Û a cos2 + b cos2 = p (định lí cosin) bc ac 2 2 Û a(1 + cos a) + B(1 + cos b) = a + b + c Û a cos A + b cos B = c Û sin 2 A + sin 2 B = 2 sin C Û 2sin( A + B)cos( A - B) = 2sin C Û cos( A - B) = 1 Û A = B. Vậy tam giác ABC cân. Baøi 6. (ĐH 2004A) Cho tam giác ABC không tù, thỏa mãn điều kiện: trong đó BC = a, CA = b, AB = c, p = cos 2 A + 2 2 cos B + 2 2 cos C = 3 Tính ba góc của tam giác ABC. ĐS: Gọi M = cos 2 A + 2 2 cos B + 2 2 cos C - 3 B +C B -C = 2 cos2 A - 1 + 2 2.2 cos .cos - 3. 2 2 A B -C A Do sin > 0, cos £ 1 nên M £ 2 cos2 A + 4 2 sin - 4. 2 2 2 Mặt khác tam giác ABC không tù nên cos A ³ 0, cos2 A £ cos A. Suy ra: M £ 2 cos A + 4 2 sin æ A Aö A - 4 = 2 ç 1 - 2sin 2 ÷ + 4 2 sin - 4 2 2ø 2 è 2 æ A A A ö = -4sin + 4 2 sin - 2 = -2 ç 2 sin - 1÷ £ 0. 2 2 2 ø è 2 Vậy M £ 0. Theo giả thiết: Trang 110 Đề thi Tốt nghiệp – Đại học ìcos2 A = cos A ï ìï A = 90o B -C M = 0 Û ïcos =1 Û í o í 2 ïî B = C = 45 ï A 1 ïsin = 2 î 2 Baøi 7. (ĐH 2004B–db2) Cho tam giác ABC thoả mãn A £ 90 0 và sin A = 2sin B sin C tan Tìm giá trị nhỏ nhất của biểu thức S = 1 - sin sin B A 2. ĐS: Chân thành cảm ơn các bạn đồng nghiệp và các em học sinh đã đọc tập tài liệu này. ncnhutqnam@gmail.com Trang 111 A . 2 [...]... d) lẻ e) lẻ f) khơng chẵn, khơng lẻ Bài 4 Giả sử y = f(x) là hàm số xác định trên tập đối xứng D Chứng minh rằng: 1 a) Hàm số F ( x ) = [ f ( x ) + f (- x )] là hàm số chẵn xác định trên D 2 1 b) Hàm số G( x ) = [ f ( x ) - f (- x )] là hàm số lẻ xác định trên D 2 c) Hàm số f(x) có thể phân tích thành tổng của một hàm số chẵn và một hàm số lẻ d) y = Bài 5 Cho hàm số y = ax 2 + bx + c (P) · Tìm a, b,... x + 6 và đường thẳng y = m Bài 6 Vẽ đồ thị của các hàm số sau: a) y = x 2 - 2 x + 1 b) y = x ( x - 2 ) c) y = x 2 - 2 x - 1 ìï- x 2 - 2 ì-2 x + 1 ì2 x nếu x ³ 0 khi x < 0 nếu x < 1 d) y = í 2 e) y = í 2 f) y = í 2 ïỵ2 x - 2 x - 3 nếu x ³ 1 ỵ x + 4 x + 1 nếu x < 0 ỵ x - x khi x ³ 0 Trang 14 Hàm số bậc nhất – bậc hai BÀI TẬP ƠN CHƯƠNG II Bài 1 Tìm tập xác định của các hàm số sau: a) y = 2 - x d) y =...Hàm số bậc nhất – bậc hai Nguy n Cơng Nh t Bài 2 Xét tính chẵn lẻ của các hàm số sau: a) y = x + 2 - x - 2 c) y = 2 x 2 - x b) y = 2 x + 1 + 2 x - 1 x +1 + x -1 e) y = 2 x + 1 x +1 - x -1 ĐS: Chẵn: b, c Lẻ: a, d Bài 3 Xét tính chẵn lẻ của các hàm số sau: d) y = a) y = x + 2 + 2 - x b) y = x + 2 - 2 - x d) y = x + 3 + x - 3 e) y = x + 2 - 2 - x + ĐS: Chẵn: Lẻ: Trang 10 c) y = x + 3 + - x + 3 3 x Hàm số. .. bằng –1 ĐS: Trang 13 Hàm số bậc nhất – bậc hai Nguy n Cơng Nh t Bài 4 Chứng minh rằng với mọi m, đồ thị của mỗi hàm số sau ln cắt trục hồnh tại hai điểm phân biệt và đỉnh I của đồ thị ln chạy trên một đường thẳng cố định: m2 a) y = x - mx + -1 4 ĐS: 2 b) y = x 2 - 2mx + m 2 - 1 Bài 5 Vẽ đồ thị của hàm số y = - x 2 + 5 x + 6 Hãy sử dụng đồ thị để biện luận theo tham số m, số điểm chung của parabol... 3) x - m + 1 ĐS: Bài 10 Vẽ đồ thị của các hàm số sau: ì- x ì-2 x - 2 khi x £ -1 khi x < -1 ï ï a) y = í1 khi - 1 < x < 2 b) y = í0 khi - 1 £ x £ 2 ïỵ x - 1 khi x ³ 2 ïỵ x - 2 khi x ³ 2 1 5 c) y = 3x + 5 d) y = -2 x - 1 e) y = - 2 x + 3 + 2 2 f) y = x - 2 + 1 - x g) y = x - x - 1 h) y = x + x - 1 + x + 1 Trang 12 Hàm số bậc nhất – bậc hai III HÀM SỐ BẬC HAI y = ax 2 + bx + c (a ¹ 0) · Tập xác định: D... y = x + 2 - 2 - x + ĐS: Chẵn: Lẻ: Trang 10 c) y = x + 3 + - x + 3 3 x Hàm số bậc nhất – bậc hai II HÀM SỐ BẬC NHẤT 1 Hàm số bậc nhất y = ax + b (a ¹ 0) · Tập xác định: D = R · Sự biến thiên: + Khi a > 0, hàm số đồng biến trên R + Khi a < 0, hàm số nghịch biến trên R · Đồ thị là đường thẳng có hệ số góc bằng a, cắt trục tung tại điểm B(0; b) Chú ý: Cho hai đường thẳng (d): y = ax + b và (d¢): y = a¢x... a¢ và b = b¢ + (d) cắt (d¢) Û a ¹ a¢ 2 Hàm số y = ax + b (a ¹ 0) ì b khi x ³ ï ax + b a y = ax + b = í b ï-(ax + b) khi x < ïỵ a Chú ý: Để vẽ đồ thị của hàm số y = ax + b ta có thể vẽ hai đường thẳng y = ax + b và y = –ax – b, rồi xố đi hai phần đường thẳng nằm ở phía dưới trục hồnh Bài 1 Vẽ đồ thị của các hàm số sau: a) y = 2 x - 7 b) y = -3 x + 5 c) y = Bài 2 Tìm toạ độ giao điểm của các cặp đường... ĐS: Bài 7 Với giá trị nào của m thì hàm số sau đồng biến? nghịch biến? a) y = (2m + 3) x - m + 1 b) y = (2m + 5) x + m + 3 c) y = mx - 3 - x d) y = m( x + 2) ĐS: Bài 8 Tìm các cặp đường thẳng song song trong các đường thẳng cho sau đây: x a) 3y - 6 x + 1 = 0 b) y = -0,5 x - 4 c) y = 3 + d) 2 y + x = 6 2 e) 2 x - y = 1 f) y = 0,5 x + 1 ĐS: Bài 9 Với giá trị nào của m thì đồ thị của các cặp hàm số sau... 0 VẤN ĐỀ 3: Một số bài tập áp dụng định lí Vi–et 1 Biểu thức đối xứng của các nghiệm số b c Ta sử dụng cơng thức S = x1 + x2 = - ; P = x1 x2 = để biểu diễn các biểu thức đối a a xứng của các nghiệm x1, x2 theo S và P Ví dụ: x12 + x22 = ( x1 + x2 )2 - 2 x1 x2 = S 2 - 2 P x13 + x23 = ( x1 + x2 ) éë( x1 + x2 )2 - 3 x1 x2 ùû = S(S 2 - 3P ) 2 Hệ thức của các nghiệm độc lập đối với tham số Để tìm hệ thức... ïỵ5x - 7 xy - 6 y = 0 Bài 8 Trang 33 ìï x 2 - 4 xy + y 2 = m c) í 2 ïỵ y - 3 xy = 4 Phương trình bậc nhất – bậc hai Nguy n Cơng Nh t BÀI TẬP ƠN CHƯƠNG III Giải và biện luận các phương trình sau: Bài 1 a) m 2 x + 4m - 3 = x + m2 b) (a + b)2 x + 2a2 = 2a(a + b) + (a2 + b2 ) x c) a2 x + 2ab = b2 x + a2 + b2 ĐS: d) a(ax + b) = 4ax + b2 - 5 Tìm m để các phương trình sau có nghiệm: Bài 2 m2 x b) - m x = ... chữ số đgl chữ số (hay đáng tin) d khơng vượt q nửa đơn vị hàng có chữ số Nhận xét: Tất chữ số đứng bên trái chữ số chữ số Tất chữ số đứng bên phải chữ số khơng chữ số khơng Bài a) Trang Hàm số. .. Qui tròn số gần · Nếu chữ số sau hàng qui tròn nhỏ ta việc thay chữ số chữ số bên phải số · Nếu chữ số sau hàng qui tròn lớn hay ta thay chữ số chữ số bên phải số cộng thêm đơn vị vào chữ số hàng... + = Bài Trong tập hợp sau, tập tập tập nào? a) A = {1, 2, 3} , B = { x Ỵ N x < 4} , C = (0; + ¥) , D = { x Ỵ R x - x + = 0} b) A = Tập ước số tự nhiên 6; B = Tập ước số tự nhiên 12 c) A = Tập

Ngày đăng: 23/10/2015, 20:09

Từ khóa liên quan

Mục lục

  • DS10 C1A.doc

  • DS10 C2A.doc

  • DS10 C3A.doc

  • DS10 C3B.doc

  • DS10 C4A.doc

  • DS10 C4B.doc

  • DS10 C5A.doc

  • DS10 C6A.doc

  • DS10 C6B.doc

  • DS10 C6C.doc

  • ds10 phuluc 1.doc

  • ds10 phuluc 2.doc

  • ds10 phuluc 3.doc

Tài liệu cùng người dùng

Tài liệu liên quan